Download as pdf or txt
Download as pdf or txt
You are on page 1of 79

PHYSICS

Solution to Module 7A

Topics Page no.


Chapter 9: Ray Optics and Optical Instruments

9.1 Reflection of Light by a Spherical Mirror 1

9.2 Refraction at a Plane Surface 6

9.3 Refraction through a Prism 16

9.4 Refraction at a Spherical Surface 22

9.5 Optical Instruments 33

Chapter 10: Wave Optics

10.1 Huygen’s Principle, Doppler Effect and Interference 42

10.2 Diffraction 57

10.3 Polarisation 64

Test Paper 70
Copyright
The copyright for the material contained in this book is held by BASE Educational Services Pvt. Ltd. No part of the book may be
reproduced, stored in a retrieval system or transmitted in any form or by any means mechanical, electronic, photocopying, recording,
or otherwise, by anyone, without prior written permission from BASE Educational Services Pvt. Ltd. 27, Bull Temple Road,
Basavanagudi, Bangalore 560 004. Violators are liable for prosecution.
Note:
This courseware is designed to provide accurate and authoritative information with regard to the subject matter covered. The
contents of this courseware have been checked for accuracy. Since deviations cannot be precluded entirely, and also as this
courseware is intended for educational purpose, BASE Educational Services Pvt. Ltd. shall not be responsible for any errors,
omissions or damages arising out of the use of the information contained in this book.
9. Ray Optics and Optical Instruments
9.1 Reflection at a Spherical Surface
Exercise
Very short answer questions (VSA)
1. A ray of light falls on a mirror normally. What are the values of the angle of incidence and the angle of
reflection?
Solution
Angle of incidence = 0°, Angle of reflection = 0°
2. What is the focal length (or radius of curvature) of a plane mirror?
Solution
Infinity.
3. Which spherical mirror always forms a virtual, erect and diminished image of an object?
Solution
Convex mirror.
4. Which spherical mirror can form enlarged image?
Solution
Concave mirror.
5. What is the relation between the focal length (f) and the radius of curvature (R) of a spherical mirror?
Solution
R
f=
2
6. How the focal length of a spherical mirror gets affected, when it is placed inside water?
Solution
There will be no change in the focal length. (The focal length (f) of a spherical mirror does not depend
upon the medium in which it is placed.)

Short answer questions (SA - 2 M)


7. What is the nature of the image and its position, when an object is placed within the principal focus of a
concave mirror?
Solution
Enlarged and erect, which appears to be behind the mirror.

Short answer type questions (SA - 3 M)


8. What is the difference between virtual images formed by a plane mirror, a concave mirror and a convex
mirror?
Solution
The virtual image produced by a plane mirror is of the same size as that of the object. However, a virtual
image produced by a concave mirror is magnified, whereas that produced by a convex mirror is
diminished.
9. An object is placed on the principal axis of a concave mirror of focal length 10 cm at a distance of
8.0 cm from the pole. Find the position and the nature of the image. [40 cm, virtual]
Solution
Here, u = − 8.0 cm and f = − 10 cm

1
2PBBBCPM7AS

1 1 1
We have, + =
v u f
1 1 1 1 1 1
or, = − = − =
v f u −10 cm −8.0 cm 40 cm
C F O P
or, v = 40 cm
The positive sign shows that the image appears to be at
40 cm from the pole on the other side of the mirror.
The reflected rays do not converge and appear to come from behind the mirror.
Hence, the image is virtual.
10. An object of height 2 cm is placed on the principal axis of a concave mirror at a distance of 12 cm from
the pole. If the image is inverted, real and of height 5.0 cm, find the location of the image and the focal
length of the mirror. [30 cm, 8.6 cm]
Solution
v h
We know that, magnification m = − = I
u hO
−5.0 cm −v
= or, v = −30 cm.
2.0 cm −12 cm
The image is formed at 30 cm from the pole on the side of the object.
1 1 1 1 1 7 60
We have, = + = + =− or, f = − = −8.6 cm.
f v u −30 −12 60 7
11. Derive the relation between focal length and radius of curvature of a concave mirror. [June 2019]
Solution
Consider a concave mirror of focal length f and of radius of curvature R. Let P represent its pole, F
represent its principal focus and C its centre of to the principal axis of the concave mirror (of small
aperture) is incident at M. The ray is reflected along MF.

A M
q
q AM – incident ray
MF – reflected ray
q 2q C – Centre of curvature
C F P
F – Principal focus
f
P – Pole
CM – Normal to mirror at M.
R

Reflection from a concave mirror

From the law of reflection, angle of reflection = angle of incidence.


∴ ∠CMF = ∠AMC = θ
From the above figure ∠MCF = ∠AMC = θ (alternate angles)
Also, ∠MFP = ∠CMF + ∠MCF (Exterior angle = sum of the interior opposite angles)
As the aperture is small and angle θ is small, arc PM may be considered as a straight segment.
PC = −R, PF = −f (by sign convention)

2
2PBBBCPM7AS

PM PM   tan θ  
From ∆le PMC, tan θ = θ= ∵ lim   = 1  tan θ ≃ θ, θ is in radian  .
PC −R  θ→0  θ  
PM PM
From ∆le PMF, tan 2θ =  2θ =
PF −f
 PM 
θ  −R  R
Hence, =  or f=
2θ  PM  2
 −f 

Long answer questions (5 M)


12. Derive the mirror formula for a concave mirror forming a real image.
Solution
Refer Module 7A
13. Define magnification. Derive the expression for magnification produced by a concave mirror.
Solution
Refer module 7A

Class work Problems


14. A rod of length 10 cm lies along the principal axis of a concave mirror of focal length 10 cm in such a
way that the end closer to the pole is 20 cm away from it. Calculate the length of the image. [5 cm]
Solution
The radius of curvature of the mirror is R = 2 f = 20 cm.
Thus, the nearer end B of the rod AB is at the centre of
the curvature and hence, its image will be formed at B
itself. We shall now locate the image of A. A B
C F
Here, u = −30 cm and f = −10 cm. P
1 1 1 B’ A’
+ =
We have v u f 10 cm 20 cm
1 1 1 1 1
or, = − = − or, v = −15 cm
v f u −10 −30
Thus, the image of A is formed at 15 cm from the pole. The length of the image is, therefore, 5 cm.
15. A small candle, 2.5 cm in size is placed at 27 cm in front of a concave mirror of radius of curvature
36 cm. At what distance from the mirror should a screen be placed in order to obtain a sharp image?
Describe the nature and size of the image. [−54 cm, real, inverted, magnified twice]
Solution
Radius of curvature of the concave mirror, R = −36 cm
R −36
∴ Focal length, f = = = −18 cm
2 2
C F P
Distance of the object, u = − 27 cm
36 cm
Height of the object, hO = 2.5 cm
Use the mirror formula
1 1 1 1 1 1
= + − = −
f v u 18 v 27

3
2PBBBCPM7AS

1 1 1 −3 + 2 1
=− + = =−
v 18 27 54 54
∴ v = – 54 cm
Distance of the screen from the mirror, v = − 54 cm
Let the size of image be hI. By using the formula for magnification due to a spherical mirror,
v h
we have m = − = I
u hO
−(−54) h I
=
−27 2.5
hI = −5 cm
The negative sign shows that the image is formed in front of the mirror and it is inverted. Thus, the
screen should be placed at a distance of 54 cm and the size of image is 5 cm. It is real, inverted and
magnified.
16. A square wire of side 3.0 cm is placed 25 cm away from a concave mirror of focal length 10 cm. What is
the area enclosed by the image of the wire? The centre of the wire is on the axis of the mirror with its
two sides normal to the axis. [4 cm2]
Solution
Given ho = 3.0 cm; u = −25 cm ; f = − 10 cm
Using mirror formula we have
1 1 1 1 1 1 1 1 1 1 3
+ = or = − = − =− + =−
u v f v f u −10 −25 10 25 50
50
∴ v=− cm
3
50

Now, magnification m = h I = − v or h I = − 3
hO u 3.0 −25
 2
∴ h I = 3.0 ×  −  = −2.0 cm ∴ Side of the image of the square wire is 2 cm.
 3
Area enclosed by the image of the wire = 2 × 2 = 4 cm2
17. An object is placed at a distance of 40 cm from a convex spherical mirror of radius of curvature 20 cm.
At what distance from the object should a plane mirror be placed so that the image in the spherical
mirror and the plane mirror are in one plane? [24 cm]
Solution B B’
1 1 1
For a convex mirror + = M P
v u f
Here u = − 40 cm, f = + 10 cm, v = ?
1 1 1 A A’
∴ + = or v = + 8 cm 40 cm v
v −40 10
The distance between the object and its convex mirror image = 40 + 8 = 48 cm
Since in a plane mirror the image is as much behind the mirror as the object is in front of it, the distance
48 cm
of the plane mirror from the object must be = 24 cm.
2

4
2PBBBCPM7AS

Practice Exercise
18. The image of an object due to a convex mirror is 4 cm from the mirror. If the mirror has a radius of
curvature of 24 cm, find (i) object position (ii) the magnification. [(i) 6 cm, (ii) 2/3]
Solution
The image due to a convex mirror is always virtual and is formed behind the mirror.
R 24
Therefore, v = +4 cm. The focal length of the mirror = = = 12 cm .
2 2
Since, it is a convex mirror, f = + 12 cm
1 1 1
(i) Now, + =
u v f
1 1 1
+ = ∴ u = −6 cm
u 4 12
Since, u is negative, the object is in front of the mirror.
v  4  2
(ii) Magnification, m = − = −   =
u  −6  3
The image is two-thirds of the size of the object.
19. An object is placed at (i) 10 cm, (ii) 5 cm in front of a concave mirror of radius of curvature 15 cm. Find
the position, nature and magnification of the image in each case.
[(i) −30 cm, magnified, real and inverted, −3. (ii) 15 cm, magnified, virtual and erect, 3]
Solution
R −15
Given, R = −15 cm  f = = = −7.5 cm
2 2
(i) When u = −10 cm
1 1 1 1 1 1 1 1 10 × 7.5
Using, + =  = − = − or v = = −30 cm
v u f v f u −7.5 (−10) −2.5
The image is 30 cm from the mirror on the same side as the object is,
v (−30)
Also, m = − = − = −3
u (−10)
The image is magnified, real and inverted
(ii) When u = −5 cm,
1 1 1 5 × 7.5
Using, + =  v = = 15 cm
v u f (7.5 − 5)
The image appears to be formed at 15 cm behind the mirror. It is a virtual image.
v −15
Also, m = − = =3
u (−5)
The image is magnified, virtual and erect.
20. An erect image three times the size of the object is obtained with a concave mirror of radius of curvature
36 cm. What is the position of the object? [12 cm]
Solution
An erect image w.r.t. object is obtained with a concave mirror, only when the image is virtual. Therefore,
image is formed behind the mirror and magnification is positive. If x is the distance of the object from
R 36
the mirror, then u = − x ; v = +3x and f = − = − = −18 cm
2 2

5
2PBBBCPM7AS

1 1 1
Now, from mirror formula + =
u v f
1 1 1 −2 1
or + = or =−
− x 3x −18 3x 18
∴ x = 12 cm i.e., u = − 12 cm
Thus, the object is at 12 cm in front of the mirror.
21. A 4.5 cm needle is placed 12 cm away from a convex mirror of focal length 15 cm. Obtain the location
5
of the image and the magnification. [6.7 cm, ]
9
Solution
Focal length of convex mirror, f = + 15 cm B
Distance of the object u = − 12 cm
A P F C
Size of the object, hO = 4.5 cm
1 1 1
Using the mirror formula, + =
v u f
1 1 1 1 1 1 9 20
+ = , = + = ∴v= ≃ 6.7 cm
v (−12) 15 v 15 12 60 3
Distance of the image from the mirror v = 6.7 cm.
The positive sign shows that the image appears to be formed behind the mirror (virtual).
v h
Using the formula for magnification, m = − = I
u hO
 6.7  h I
− =
 −12  4.5
Size of the image hI = 2.5 cm
As hI is positive, image is erect and virtual.
h 2.5 5
Magnification m is given by m = I = =
h O 4.5 9

9.2 Refraction at a Plane Surface


Exercise
Very short answer questions (VSA)
1. What is an optical medium?
Solution
A medium which allows light to propagate through it is called an optical medium. (It may be a material
medium or even vacuum).
2. Define isotropic medium.
Solution
A medium is isotropic if the speed of light is same in all directions.
3. What is the meaning of the statement refractive index of water is (4/3)?
Solution
The ratio of sine of the angle of incidence in air to the sine of the angle of refraction in water is (4/3).
Aliter: The ratio of velocity of light in vacuum (or air) to that in water is (4/3).

6
2PBBBCPM7AS

4. Define absolute refractive index of a medium.


Solution
The ratio of speed of light in vacuum to the speed of light in a given medium is called absolute refractive
index of that medium.
5. Define relative refractive index of a medium.
Solution
Refractive index of medium-2 with respect to medium-1 is defined as the ratio of the speed of light in the
v
first medium to that in the second medium i.e., n 21 = 1 .
v2
Aliter
Refractive index of medium-2 with respect to medium-1 is defined as the ratio of wavelength of light in
λ
first medium to that in the second medium. i.e., n 21 = 1 .
λ2
6. What is the limitation of Snell’s law?
Solution
Snell’s law is not valid for normal incidence.
7. What is angle of deviation?
Solution
The difference between the angle of incidence and the angle of refraction is called the angle of deviation
(d = i ~ r)
8. A ray of light passes from glass of refractive index 3/2 to water of refractive index 4/3. Calculate the
(i) refractive index of glass with respect to water
(ii) refractive index of water with respect to glass
(iii) the deviation of the ray if the angle of incidence is 30°
(iv) ratio of the velocities of light in water and glass
Solution
n g (3 / 2) 9
(i) n gw = = = = 1.125
n w ( 4 / 3) 8
n w (4 / 3)
(ii) n wg = = = 0.88
n g (3 / 2)
sin i n sin(30°) (3 / 2)
(iii) n wg =  w =  sin r = × 0.5  r = 34°13′
sin r ng sin r (4 / 3)
d = i ~ r = 34°13′ ~ 30° = 4°13′
v n g (3 / 2) 9
(iv) w = = =
vg n w (4 / 3) 8
9. Define lateral shift.
Solution
When a ray of light is incident obliquely on a parallel sided refracting slab, surrounded by a homogenous
medium, the ray experiences equal but opposite deviations at the two parallel faces. Thus, the emergent
ray is parallel to the incident ray. The perpendicular distance between the direction of incident ray and
the emergent ray is called lateral shift.

7
2PBBBCPM7AS

10. Define normal shift.


Solution
Normal shift is defined as the distance through which an object appears to be shifted when placed in one
medium and is observed from another medium (of different refractive index), observation being made
along the normal to the surface separating the two media.
11. Define critical angle for a pair of media.
Solution
Critical angle for a pair of media (for a given wavelength) is that angle of incidence in the denser
medium for which the angle of refraction is 90°, i.e., the refracted ray just grazes the interface of the two
media.
12. Write the relation between critical angle and refractive index.
Solution
Refractive index of rarer medium
sin C =
Refractive index of denser medium
13. What is the principle on which an optical fibre works?
Solution
It works on the principle of total internal reflection.
14. Mention an application of optical fibre.
Solution
Optical fibre communication.

Short answer questions (SA - 2 M)


15. Snell’s law fails to give the value of refractive index for normal incidence. Justify.
Solution
sin i
According to Snell’s law, n 21 = .
sin r
For normal incidence there is no bending i.e., for i = 0, r = 0
sin i 0
∴ n 21 = = which is an indeterminate.
sin r 0
n should have a finite value.
16. State the laws of refraction.
Solution
First law
The incident ray, the refracted ray, and the normal to the refracting surface at the point of incidence lie in
the same plane.
Second law (Snell’s law of refraction)
The ratio of the sine of the angle of incidence to the sine of the angle of refraction is a constant, for a
given pair of media and for a given wavelength of light.
sin i
i.e., = constant
sin r
This is known as Snell’s law.

8
2PBBBCPM7AS

17. Distinguish between relative and absolute refractive indices.


Solution
Relative refractive index of medium 2 w.r.t. medium 1 is the ratio of the velocity of light in
v1
medium 1 (v1) to the velocity of light in medium 2 (v2) n 21 = . Relative refractive index can be greater
v2
than 1 or less than 1.
Absolute refractive index of a medium is the ratio of the velocity of light in vacuum (c) to the velocity of
c
light in the given medium (v) n = . Absolute refractive index is always greater than 1.
v
18. For which colour of light normal shift is (i) maximum (ii) minimum
Solution
 1
We know that SN = t 1 −  . For a given real depth (t), since relative refractive index (n21) of a pair of
 n
media is greatest for violet and least for red, normal shift is maximum for violet and least for red.
19. Write the conditions for total internal reflection to take place.
Solution
1. Light should tend to travel from an optically denser medium to an optically rarer medium.
2. The angle of incidence in the optically denser medium must be greater than the critical angle for the
given pair of media, for a given wavelength.
20. Mention any two applications of total internal reflection. [March 2014]
Solution
1. Sparkling of diamond
2. Mirage
3. Total reflecting prism

Short answer type questions (SA - 3 M)


21. A ray of light incident at 60° on a glass slab is partially reflected and partially refracted. If the angle
between the reflected and refracted rays is 90°, find the refractive index of the glass.  3 
Solution
r + i = 90°  r = 90° – i

air i i
90°
r

Glass slab

3
sin i sin 60° sin 60° 2 = 3
∴n= n= n= ,
sin r sin(90° − 60°) sin 30° 1
2

9
2PBBBCPM7AS

22. A ray of light travelling in water is incident at an angle of 30° on a glass surface. Calculate the angle of
refraction and the angle of deviation. Given: refractive index of water is 4/3 and that for glass is 3/2.
[26° 23′, 3° 37′]
Solution
sin i n g sin i n
n gw =  =  sin r = w sin i
sin r n w sin r ng
4
sin r = 3 × sin(30°)  r = 26°23′  d = i ~ r = 30° − 26°23′ = 3°37′
3
2
23. Monochromatic light of wavelength 589 nm is incident on air-water interface. Calculate the wavelength,
frequency and velocity of (i) reflected and (ii) refracted light. Refractive index of water is 1.33.
[(i) 589 nm, 5.09 ×1014 Hz , 3 × 108 m s–1 (ii) 442.857 ×10-9 m , 5.09 × 1014 Hz, 2.254 ×108 m s-1 ]
Solution
(i) For reflected light, λ = 589 nm
Since, v = f λ
v 3 × 108
f= = −9
= 5.09 × 1014 Hz.
λ 589 × 10
v = 3 × 108 m s–1
(ii) For refracted light
n air λ n
= water  λ water = λ air × air
n water λ air n water
1
λ water = 589 × 10−9 × = 442.857 × 10−9 m
1.33
Frequency of light does not change on passing from one medium to another.
14
∴ Frequency, f = 5.09 × 10 Hz
v w = fλ w = 5.09 × 1014 × 442.857 × 10−9
v w = 2.254 × 108 m s −1
24. Sun is seen a little before it rises and for a short while after it sets. Explain.
Solution
Because of atmospheric refraction, the apparent shift in the direction of the sun is by about half a degree
and the corresponding time difference between apparent sunrise and the actual sunrise is about two
minutes. Thus, the sun is visible roughly two minutes earlier than the actual sunrise (Actual sunrise
means the actual crossing of the horizon by the sun). Similarly, the time difference between the actual
sunset and apparent sunset is also about two minutes. Thus, sun is seen for a short duration of time even
after the sunset.
25. What are the factors on which the normal shift depends?
Solution
Normal shift depends on
(i) distance between the interface and the position of the object.
(ii) relative refractive index of the medium in which the object is situated.

10
2PBBBCPM7AS

26. Obtain the relation between critical angle and refractive index for a pair of media.
Solution
In the figure, XY is a surface separating two media of refractive indices (for a given wavelength) n1 and
n2 (n1 > n2). Let C be the critical angle for the given pair of media. Consider a ray IO incident at an angle
C in medium–1. It is refracted as OR grazing the surface of separation XY.
Now i = C and r = 90° M
Medium-2
By Snell’s law, n1 sin i = n 2 sin r (Rarer - n2)
n1 sin C = n 2 × 1 (∵ sin 90° = 1) O
X Y
n2 Refractive index of rarer medium (n r ) R
∴ sin C = = Medium-1
n1 Refractive index of denser medium (n d ) (Denser - n1)
i=C
n1 > n2
nr 1 I
∴ sin C = . Thus, n dr =
nd sin C N

For i > C, Snell’s law is not satisfied. Thus, no refraction Refractive index and critical angle
If the rarer medium is air or vacuum, n dr = n = absolute refractive index of the medium,
1
Then, n = where, C is the critical angle for the pair of media.
sin C
27. Mention some optical instruments which use total reflection prisms.
Solution
Submarine periscope, binoculars, camera view finders, reflecting telescopes.
28. Write a note on optical fibre with respect to construction and working.
Solution
An optical fibre consists of a core of higher refractive index surrounded by cladding of lower refractive
index. It works on the principle of total internal reflection. When a ray of light entering the fibre from
one end is incident on the core-cladding interface at an angle greater than the critical angle for the fibre,
it undergoes total internal reflection several times before emerging out at the other end of the fibre with
negligible loss of light energy.
29. What is total internal reflection? Mention any two applications of the optical fibres.
Solution
When a ray of light travels from a denser to a
rarer medium, the refracted ray bends away from
Rarer r r = 90°
the normal. The angle of refraction is greater
i i
than the angle of incidence. As the angle of Denser i
incidence increases, the angle of refraction also i i

increases. For a particular angle of incidence, the (a) i < C (b) i = C (c) i > C
angle of refraction becomes 90° and the Total internal reflection
refracted ray grazes the surface separating the two media. This angle of incidence is called the critical
angle and is defined for a pair of media. If the angle of incidence is increased beyond the critical angle,
the ray is not refracted but gets reflected as shown in the figure. The entire incident light is reflected
back into the denser medium. This phenomenon is called total internal reflection.
Optical fibres are used in communication and in medical diagnostic tools.

11
2PBBBCPM7AS

30. Explain the phenomenon of a mirage.


Solution
A mirage is an optical illusion caused due to refraction and internal reflection.
In deserts, the layers of air closer to the sand are hotter and hence rarer than those farther away. Rays of
light from a distant object like a tree, travelling downwards, undergoes a series of refractions. At each
refraction, the rays travel from a denser to a rarer medium. Hence, they bend away from the normal.

When they reach the lower layers, they may be incident at angles greater than the critical angle. Hence,
they undergo total internal reflection. Thus, one can see an inverted image of the tree as if it is reflected
by a pool of water. Further, due to fluctuations in the density of air, the image appears to quiver, giving
an impression of reflection by a disturbed water surface. This illusion is called a mirage.

Long answer type question (LA)


31. Explain total internal reflection, with the help of a neat diagram. Obtain the relation between refractive
index and critical angle.
Solution
Refer module 7A
Classwork Problems
32. A small pin fixed on a table top is viewed from above from a distance of 50 cm. By what distance would
the pin appear to be raised if it is viewed from the same point through a 15 cm thick glass slab held
parallel to the table? Refractive index of glass = 1.5. Does the answer depend on the location of the slab?
[5 cm, No]
Solution
Given, thickness of glass slab (real depth) = 15 cm
Refractive index of glass ang with respect to air = 1.5
Real depth 15
n ga = =
Apparent depth y
Apparent depth of the pin, y = 15/1.5 = 10 cm
Distance by which the pin appears to be raised = Real depth − Apparent depth = 15 − 10 = 5 cm
The distance through which the pin appears to be raised does not depend on the location of the slab,
because the distance travelled by light in glass remains same.

12
2PBBBCPM7AS

Alternate Method
Thickness of glass slab, t = 15 cm
Refractive index of glass ng = 1.5
The normal shift in the position of the pin,
 1  2 1
SN = t  1 −  = 15 1 −  = 15 × = 5 cm
 n  3 3
The pin appears to be raised by 5 cm.
33. A small air bubble is trapped in a glass cube of edge 0.24 m. When viewed from one face, it appears to
be 0.1 m from the surface and when viewed through the opposite face it appears to be 0.06 m from the
surface. Calculate the refractive index of glass. [1.5]
Solution
Let the real position of the air bubble from face–1 be x m and the apparent depth from this face is 0.1 m.
Real depth x
From definition, n = = …(1)
Apparent depth 0.1 m
From the face-2 real depth = (0.24 – x)
Apparent depth = 0.06 m
realdepth 0.24 − x 0.24 m
∴n = = …(2)
apparent depth 0.06 Face-1 x O
Face-2
from Eqs., (1) and (2) ( 0.24 − x )
x 0.24 − x
=  0.06x = 0.1 [0.24 − x]
0.1 0.06
0.06x = 0.024 – 0.1x  0.16x = 0.024 ∴ x = 0.15 m
0.15
From Eq. (1) n = = 1.5 ∴ refractive index of glass = 1.5
0.1
34. A small bulb is placed at the bottom of a tank containing water to a depth of 80 cm. What is the area of
the surface of water through which light from the bulb can emerge out? Refractive index of water is
1.33. (Consider the bulb to be a point source.) [2.58 m2]
Solution
Let the bulb be placed at point O. AB = AC = r
(If the light falls at an angle of incidence equal to critical angle ic then
only a circular area is formed because if angle of incidence is less than the r A r
C B
critical angle it will refract into air and when the angle of incidence is i c i c

greater than critical angle then it will be reflected back into water.) i i c c

The source of light is 80 cm below the surface of water i.e.


O
AO = 80 cm, nw = l.33, d = 80 cm
Using the formula for critical angle, represented as ic
1 1
sin i c =  sin ic = = 0.75
nw 1.33
ic = 48.6°
AB
tan ic =
AO
r
In ∆OAB, or tan i c =
d

13
2PBBBCPM7AS

r = d tan ic = 80 tan (48.6º)


r = 80 × 1.1345 = 90.7 cm
Area of the circular surface of water through which light will emerge is, A = π r2
A = 3.14 × (90.7)2 = 25865.36 cm2  A = 2.58 m2

Practice Exercise
35. A tank is filled with water to a height of 12.5 cm. The apparent depth of a needle lying at the bottom of
the tank is measured by a microscope to be 9.4 cm. What is the refractive index of water? If water is
replaced by a liquid of refractive index 1.63 up to the same height, by what distance would the
microscope have to be moved to focus on the needle again? [1.33, 1.73 cm]
Solution
Case I: The tank is filled with water.
Apparent depth = 9.4 cm
Height of water, t = 12.5 cm  Real depth = 12.5 cm
Realdepth 12.5
Refractive index of water n w = = = 1.33
Apparent depth 9.4
Case II: The tank is filled with the liquid.
Refractive index of liquid n = 1.63
Real depth
n liq =
Apparent depth
12.5
 1.63 =
Apparent depth
12.5
Apparent depth = = 7.67 cm
1.63
∴ The microscope is to be shifted by a distance equal to the difference between the apparent
depths.
∴ Shift in the microscope = 9.4 − 7.67 = 1.73 cm
36. Calculate the critical angle for a glass-air surface, if a ray of light which is incident in air on the surface
is deviated through 15° when the angle of incidence is 40°. [41° 8′]
Solution
i = 40°, d = 15°, C = ?
d = i − r  r = i − d  r = 40° − 15°  r = 25°
sin i sin(40°)
n= = = 1.52
sin r sin(25°)
Refractive Index of rarer medium
sin C =
Refractive Index of denser medium
1
sin C = = 0.6578  C = 41°8′
1.52

14
2PBBBCPM7AS

37. The velocity of light in the medium A is 2 × 108 ms–1 and velocity of light in the medium B is
2.5 × 108 m s–1. Calculate the critical angle for A-B interface. [53° 07′]
Solution
Refractive Index of rarer medium n
sin C = = r
Refractive Index of denser medium n d
c
v v
sin C = r = d
c vr
vd
2 × 108
 sin C =  C = 53°7′
2.5 × 108
38. For a fish in water, outside world would appear to be contained within a circle of area 12.56 m2 at the
surface. If the refractive index of water is 4/3, find the depth at which the fish is present. [1.76 m]
Solution
Area = πR2  12.56 m2 , R = 2 m, r = C
R R
From the figure sin C = sin r = 2 2
…(1)
R +h
r h
Refractive index of rarer medium 3
sin C = = …(2) r
Refractive index of denser medium 4
3 R
From Eqs. (1) and (2) =
4 R + h2
2

9 R2
 = 2
16 R + h 2
9R2 + 9h2 = 16R2
9h2 = 7R2
2
2 7 × ( 2)
h =
9
 h = 1.76 m
39. Figure shows a cross-section of a ‘light pipe’ made of a glass fibre of refractive index 1.68. The outer
covering of the pipe is made of a material of refractive index 1.44. What is the range of the angle of the
incident rays with the axis of the pipe for which total reflections inside the pipe take place, as shown in
the fig?
i
r i
i1
[0 < i < 60°]
Solution
Refractive index of the glass fibre (core) with respect to air, ng = n2 = 1.68
Refractive index of the outer coating material (clad) with respect to air = nclad = n1 = 1.44
Let the critical angle be C for core – clad interface.
n 1
n 21 = 2 =
n1 sin C i
r i
i
n 1.44 1
core
sin C = 1 = = 0.8571
n 2 1.68

15
2PBBBCPM7AS

C = 59°
Total internal reflection will take place when the angle of incidence i at the core – clad interface is
greater than the critical angle C.
i.e., i > 59° or when angle of refraction,
r < rmax, where rmax = 90° − ic = 90° − 59° = 31°
For air-core interface, sin i1 max = 1.68 sin 31°
= 1.68 × 0.5156
 i1 max = sin−1 (0.8662) = 60°
Thus, all the rays which are incident in the range 0 < i < 60°, will undergo total internal reflection within
the pipe.

9.3 Refraction through a Prism


Exercise
Very short answer questions (VSA)
1. What is a prism?
Solution
A prism is an optical medium bound by at least two non parallel refracting surfaces.
2. Define angle of a prism.
Solution
The angle between the refracting faces of a prism is called angle of the prism.
3. Define principal section of a prism.
Solution
A section of the prism perpendicular to its refracting edge is called principal section of the prism.
4. A glass prism is surrounded by a liquid whose refractive index is equal to that of the prism. What will be
the deviation of a ray of light passing through the prism?
Solution
There will be no deviation.
5. Write the expression for the deviation of a ray passing through a thin prism.
Solution
δ = A (n −1)
6. How does the sky appear dark to an astronaut in space?
Solution
The sky appears dark because there is no atmosphere to scatter light.
7. What is the sequence of colours in a (i) primary rainbow? (ii) secondary rainbow?
Solution
(i) Outer arc is red and the inner arc is violet.
(ii) Outer arc is violet and the inner arc is red.
8. Why clouds are white?
Solution
In the case of large scattering objects (a >> λ) like rain drops, dust, ice particles, light of all wavelengths
are scattered almost equally. Thus, the clouds appear white. (This is Mie scattering)

16
2PBBBCPM7AS

Short answer questions (SA - 2 M)


9. Show the path of a monochromatic light ray through a prism in the minimum deviation position.
Solution
In the minimum deviation position, A
i1 = i2 and r1 = r2

i1 r1 r2 i2

10. How does the angle of minimum deviation of a ray change on immersing a prism (RI = 1.5) in a medium
of lower refractive index?
Solution
When a prism initially in air is immersed under a liquid of refractive index less than that of the prism,
then the relative refractive index of the material of the prism decreases. Hence Dm decreases.
11. A ray of light incident normally on one face of a right angled prism emerges grazing the hypotenuse.
Calculate the angle of the prism. (n = 1.5). [41° 49′]
Solution
i1 = 0, r1 = 0, r2 = C and i2 = 90°
sin i 2 1
n=  1.5 =
sin r2 sin r2
1
sin r2 =  r2 = 41°49′. Since, A = r1 + r2
1.5
 A = 41° 49′
12. The angle of minimum deviation produced by an equilateral prism is equal to its refracting angle.
Calculate the refractive index of the material of the prism. [ 3]
Solution
D = A = 60°, n = ?
A+D  60° + 60° 
sin   sin  
n=  2   n=  2  = sin(60°) = 3 / 2 = 3
A  60°  sin(30°) 1
sin   sin   2
2  2 
13. Draw a neat labeled diagram showing the formation of a primary rainbow and explain at what angles the
primary rainbow is visible.
Solution
Sunlight
Raindrop
Red
Violet
Normal
o
40.8
(Violet) Violet
d
Re

o
42 (Red)

Violet

Red
Formation of primary rainbow

17
2PBBBCPM7AS

14. What are the two necessary conditions for the formation of rainbow?
Solution
(i) Sun should be at the backside of the observer
(ii) Sun, observer’s eye and the centre of the rainbow, should lie in the same plane.
15. State and explain Rayleigh’s law of scattering.
Solution
If particles are of sizes smaller than the wavelength of light, (a << λ) the intensity of scattered light
varies inversely as the fourth power of the incident wavelength.
1
I ∝ 4 . This is known as Rayleigh’s law of scattering.
λ
16. Why does the setting sun and rising sun look red?
Solution
During sunrise and sunset, light from Sun near the horizon reaches the eye in a nearly horizontal
direction. In this light, blue is scattered sideways more than red. Also, light has to pass through a longer
distance through the denser (due to dust etc) region of atmosphere. Hence, in the light received by the
eye, the proportion of blue is much less compared to red and hence the sun appears red.
Similarly, full moon near the horizontal appears reddish.
17. Why does the sky appear blue?
Solution
The blue colour of the sky may be explained on the basis of Rayleigh scattering as follows.
Sunlight travelling through earth’s atmosphere, undergoes scattering, by air molecules. The scattered
light spreads sideways, illuminating the sky. When we look at the sky, in a direction away from direct
sunlight, we see the sky by scattered light present in that region. In Rayleigh scattering, intensity of
scattered light is inversely proportional to the fourth power of wavelength. The wavelength of blue light
(~ 450 nm) is about 1.7 times smaller than that of red light (750 nm). Therefore, the intensity of blue
colour in the scattered light is about 8 (≅ (1.7)4) times greater than that of red light. Hence, the sky
appears blue.

Short answer type questions (SA - 3 M)


18. The deviations produced by a crown and a dense flint glass prisms are in the ratio 3:5. Calculate the ratio
of their angles [ncrown = 1.54, ndense flint = 1.72] [0.8]
Solution
nC = 1.54, nF = 1.72
δC 3 A C
= , =?
δF 5 A F
δC (n C − 1)A C
=
δ F ( n F − 1) A F
3 (1.54 − 1)  A C 
=  
5 (1.72 − 1)  A F 
A 3 × 0.72
∴ C = = 0.8
A F 5 × 0.54

18
2PBBBCPM7AS

19. A crown glass prism of refractive index 1.55 and a flint glass prism of refractive index 1.66 have equal
angles. Compare the deviations produced. [(5/6)]
Solution
δ
nC = 1.55, nF = 1.66, AC = AF, C = ?
δF
δC (n C − 1)A C (1.55 − 1) 5
= = =
δ F (n F − 1)A F (1.66 − 1) 6
20. Explain the formation of secondary rainbow.
Solution
R Raindrop
The formation of secondary rainbow is V
V
shown in figure. In this case, two R

refractions and two internal reflections


V
occur. The violet component has R
Sunlight
maximum angle of deviation (53.2°) 53.2
o

(violet)
whereas red component has minimum 50.1o
angle of deviation (50.1°). The width of (Red)

a secondary rainbow is about twice the


R
width of the corresponding primary V
rainbow.
Formation of secondary rainbow

21. Distinguish between primary rainbow and secondary rainbow.


Solution
Primary rainbow Secondary rainbow
1. Formed due to two refractions and one internal Formed due to two refractions and two internal
reflection. reflections.
2. Violet colour is formed at the bottom while red Red colour is formed at the bottom while violet
colour is formed at the top of the rainbow. colour is formed at the top of the rainbow.
Angle of deviation for violet is 40.8°. Angle of deviation for violet is 53.2°.
Angle of deviation for red is 42°. Angle of deviation for red is 50.1°.
3. Intensity of colours is comparatively large Intensity of colours is comparatively small as
since one reflection takes place at the back of a two reflections occur at the back of a drop.
drop

Long answer questions (5M)


22. Obtain the expression for the refractive index of the material of a prism in terms of the angle of the prism
and the angle of minimum deviation. [March 2015]
Solution
Refer Module 7A

19
2PBBBCPM7AS

Class work Problems


23. A prism of angle 60° produces angle of minimum deviation of 40°, calculate the (i) refractive index of
the material of the prism (ii) angle of incidence. [(i) 1.532 (ii) 50°] [June 2014]
Solution
Prism is in the minimum deviation position.
Given: A = 60°, D = 40°, n = ?, i = ?
A+D  60° + 40° 
sin   sin  
1. n =  2  =  2  = sin50° = 0.7660 = 1.532
A  60°  sin 30° 1
sin   sin    
2  2  2
A+D (60° + 40°)
2. i =   = = 50°
 2  2
24. A certain prism is found to produce a minimum deviation of 38°. It produces a deviation of 44° when the
angle of incidence is either 42° or 62°. Determine the refracting angle of the prism, the angle of
incidence for minimum deviation and the refractive index of the material of the prism. [60°, 49°, 1.51]
Solution
The path of a light through a prism is reversible. Thus, when deviation is same for two angles of
incidence, one of them can be taken as the angle of incidence and the other as the angle of emergence.
∴ i1 = 42° and i′ = 62°, δ = 44°, Dm = 38°
i 1 + i 2′ = A + δ
∴ 42° + 62° = A + 44  A = 60°
When deviation is minimum i1 = i2′
∴ i + i = 60° + 38°  i = 49°
A A + Dm
n sin = sin
2 2
60° + 38°
sin
∴ n= 2 = 1.51
sin 30°
25. At what angle should a ray of light be incident on the face of an equilateral prism, so that it just suffers
total internal reflection at the other face? The refractive index of the material of the prism is 1.5.
[March 2020]
Solution
27.9°
26. A ray of monochromatic light is incident normally on one face of a triangular prism of refracting angle
30° and refractive index 1.5. Find the angle of emergence and the deviation produced. [48° 36′, 18° 36′]
Solution
Since the ray is incident normally, i1 = 0°, thus, r1 = 0°
We know that r1 + r2 = A
∴ 0 + r2 = 30°  r2 = 30°
sin i 2
n=
sin r2
∴ sin i2 = 1.5 sin 30° or i2 = 48° 36′
i1 + i2 = A + D
∴ D = (i1 + i2) − A = (0 + 48° 36′) − 30° = 18° 36′

20
2PBBBCPM7AS

Practice Exercise
27. Calculate the angle of a prism made of a material of refractive index 1.618 producing a minimum
deviation of 48°. [60°]
Solution
 A + Dm 
sin  
n = 1.618 and Dm = 48° We have, n =  2 
A
sin  
2
sin ( A / 2 ) cos ( D m / 2 ) + cos ( A / 2 ) sin ( D m / 2 )
n =
sin ( A / 2 )
n = cos (Dm/2) + cot (A/2) sin (Dm/2)
1.618 = cos 24° + sin 24° cot (A/2) = 0.9135 + 0.4067 cot (A/2)
0.7045 1 1
∴ cot (A / 2) = = 1.732  tan (A / 2) = =
0.4067 1.732 3
(A/2) = 30° i.e., A = 60°
28. At what angle should a ray of light be incident on the face of a prism of refracting angle 60°° so that it
just suffers total internal reflection at the other face? The refractive index of the material of the prism is
1.524. [30°]
Solution
Angle of prism, A = 60°
Refractive index of prism material, n = 1.524
Let i be the angle of incidence. The critical angle is ic.
1 1
sin i c = = = 0.6561  ic = 41° For a prism, r1 + r2 = A
n 1.524
Here, r2 = ic ∴ r1 + ic = A
r1 +41° = 60°  r1 = 19°
sin i1
Using the formula, n =
sin r1
or sin i1 = 1.524 × sin 19° = 1.524 × 0.3256 or i1 = sin−1 (0.4962)
i1 = 29° 75′ ≈ 30°
29. A prism is made of glass of unknown refractive index. A parallel beam of light is incident on a face of
the prism. The angle of minimum deviation is measured to be 40°°. What is the refractive index of the
material of the prism? The refracting angle of the prism is 60°. [1.532, 10°20′]
Solution
Angle of minimum deviation Dm = 40°
The refracting angle of the prism A = 60°
 A + Dm   60° + 40° 
sin   sin  
Refractive index of glass w.r.t. air, n ga =  2 =  2  = sin 50°
A sin 30° sin 30°
sin
2
0.766
n ga = = 1.532
0.5

21
2PBBBCPM7AS

9.4 Refraction at a Spherical Surface


Exercise
Very short answer questions (VSA)
1. Define (i) aperture, (ii) pole, (iii) centre of curvature, (iv) radius of curvature, (v) principal axis of a
spherical surface.
Solution
(i) Aperture: The area of a spherical surface available for refraction is called its aperture.
(ii) Pole (P): The geometric centre of a spherical refracting surface is called its pole (P).
(iii) Centre of curvature (C): The centre of the sphere of which the refracting surface is a part is called
the centre of curvature (C).
(iv) Radius of curvature (R): The radius of the sphere of which the refracting surface is a part is called
its radius of curvature (R).
(v) Principal axis: The straight line passing through the pole of a spherical refracting surface and its
centre of curvature is called its principal axis.
2. What is a lens?
Solution
It is an optical medium bound by two surfaces of which at least one is either spherical or cylindrical.
3. Define principal focus of a lens.
Solution
When a parallel monochromatic paraxial beam of light is incident on a lens, after refraction, the rays
converge to a point or appear to diverge from a point on the principle axis. This point of convergence or
divergence is called principal focus.
4. Define focal length of a lens.
Solution
The distance between the optic centre of the lens and the principal focus is called as the focal length of
the lens.
5. What are paraxial rays?
Solution
Rays close to the principal axis or incident at small angles with the principal axis are called paraxial rays.
6. Define power of a spherical lens.
Solution
It is a measure of the deviation of the incident beam of light. It is (measured in dioptre) numerically
equal to the reciprocal of the focal length (in metre) of the lens.
7. A portion of a lens is broken. Can we get a complete image of an object by using a broken piece?
Solution
Yes. But, the intensity of the image will be less.
8. What is the magnification of an image produced by lens when u = 2f?
Solution
One.
9. Where do we get the image of an object, when it is placed at the principal focus of a lens?
Solution
At infinity.

22
2PBBBCPM7AS

10. When do we consider a lens as a thin lens?


Solution
When its thickness is negligible compared to the focal length of the lens.
11. A lens has a focal length of 0.1 m. What is its power?
Solution
1 1
Power P = = = 10 D
f 0.1
12. How does the power of a lens vary with its focal length?
Solution
Power decreases, as the focal length increases.
13. A converging lens made of a material of refractive index 1.5 is immersed in a liquid of refractive index
1.3. How does its focal length change?
Solution
Focal length increases. (Power decreases)
14. What is the power of a lens immersed in a liquid of refractive index equal to that of the lens?
Solution
Zero.
15. A converging lens is immersed in a liquid whose refractive index is higher than that of the lens. What
happens to the nature of the lens?
Solution
Behaves as a diverging lens.
16. A converging and a diverging lens of same focal length are in contact. What is the focal length of the
combination?
Solution
1 1 1
We know that the focal length of the combination of two thin lenses in contact is given by = + .
f f1 f 2
1
Since, f1 = −f2, = 0 . Therefore, the equivalent focal length is infinity.
f
17. Which property of an image gets affected when a portion of the lens is blocked?
Solution
The intensity of the image decreases.

Short answer type questions (SA - 2 M)


18. The percentage change in the radius of curvature of each surface of a lens is 20. Calculate the percentage
change in its focal length. [20%]
Solution
1 2
= (n − 1)  
f R
1  2 
= (n − 1)  
f ′  R′ 
f ′ R′ f′ R′ − R R′ − R
=  −1 =  × 100 = 20%
f R f R R

23
2PBBBCPM7AS

19. An object is placed at a distance of 0.1 m from a convex lens of focal length 0.2 m. Determine the nature
of the image. [−0.2 m]
Solution
u = −0.1 m, f = 0.2 m, v = ?
1 2
= (n − 1)  
f R
1 1 1 1 1 1
− =  − =  v = −0.2 m . So the image is virtual and erect.
v u f v (−0.1) 0.2
20. An object and its real image formed by a lens are at equal distances from the lens. Calculate the focal
length of the lens.
Solution
1 1 1 1 1 1 1 1 1 2 u v
= − ; Here u is −ve numerically, = +  = + = f = =
f v u f u v f u u u 2 2
1 1
21. Draw the nature of the graph when is plotted against for a lens.
v u
Solution

1
v

1
u

22. A parallel beam of red light is incident on the lens. If the beam is replaced by blue light will the principal
focus be the same? Explain.
Solution
1 1 1 
No. Since, = (n − 1)  −  , and for a given material, n is different for different wavelengths.
f  R1 R 2 
nblue > nred.
∴ f blue < f red
23. The radii of curvatures of a double concave lens are 30 cm and 60 cm, and its refractive index is 1.5.
Calculate the focal length of the lens. [−0.4 m]
Solution
We know that
1  1 1 
= (n − 1)  − 
f  R1 R 2 
Here, R1 = − 30 cm and R2 = + 60 cm, n = 1.5
1  1 1   1  1
Thus, = (1.5 − 1)  −  = 0.5 ×  −  = −
f  −30 60   20  40
or f = − 40 cm = −0.4 m

24
2PBBBCPM7AS

24. A plano-convex lens with focal length of 20 cm in air is to be ground from optical glass whose index of
refraction is 1.5. Find the radius of curvature of the curved surface. [0.1 m]
Solution
We are given that
f = 20 cm, n = 1.5 and R2 = ∞ (A plano-convex lens has one plane surface of infinite radius of curvature)
1  1 1 
As = (n − 1)  − 
f  R1 R 2 
1  1 1 1 1
= (1.5 − 1)  − = ×
20  R1 ∞  2 R1
or R1 = 10 cm = 0.1 m
25. A convex lens is made up of three different materials as shown in Fig. How many images are formed
when an object is placed on its principal axis?

Solution
Since each layer has different refractive index, three images are formed.
26. A lens of focal length f is split into two pieces as shown in the Fig. What is the focal length of each
piece?

Solution
The focal length of each piece is f.
27. An equiconvex lens of focal length f is split into two plano convex lenses as shown in
figure. What is the focal length of each plano-convex lens?

Solution
1  1 1  2(n − 1)
Before splitting: = (n − 1)  − =
f  R1 R 2  R
1  1 1  (n − 1)
After splitting: = (n − 1)  −  =
f ′ R ∞ R
Hence, f ′ = 2f
28. A converging lens of focal length 0.5 m is in contact with a diverging lens of focal length 0.25 m.
Calculate the power of the combination. [−2D]
Solution
1 1 1 1 1 1 1 1
= +  = +  =−
f eq f1 f 2 f eq 0.5 −0.25 f eq 0.5
1 1
Power, P = P=− = −2D
f 0.5

25
2PBBBCPM7AS

29. Two lenses of power 1.5 D and 2.75 D are kept in contact. Find the equivalent focal length. [0.235 m]
Solution
Peq = P1 + P2 =1.5 + 2.75
= 4.25 D
1 1
f = f = = 0.235 m
P 4.25
= 23.5 cm

Short answer type questions (SA - 3 marks)


30. What are the factors on which the focal length of a lens depends?
Solution
The focal length of a lens depends on
(a) refractive index of the material of lens.
(b) refractive index of the medium surrounding the lens.
(c) radii of curvature of the surfaces of lens.
31. Two lenses which appear to be identical are made of glass of refractive indices 1.5 and 1.6. Find the ratio
of their focal lengths. [6 : 5]
Solution
1 1 1 
= (n − 1)  − 
f  R1 R 2 
1 1 1 
= (1.5 − 1)  −  …(1)
f1  R1 R 2 
1 1 1 
= (1.6 − 1)  −  …(2)
f2  R1 R 2 
(2) f (1.6 − 1) 6
 1 = =
(1) f 2 (1.5 − 1) 5
32. Calculate the refractive index of the material of an equiconvex lens whose focal length is equal to the
radius of curvature of either surface. [1.5]
Solution
R1 = R, R2 = −R, |R1| = |R2| = |f |
1 2
= (n − 1)  
f R
1 1 1 
= (n − 1)  − 
f  R1 R 2 
1 2
= (n − 1)  
f f 
1
 (n −1) =
2
 n = 1 + 0.5 = 1.5

26
2PBBBCPM7AS

33. The image of a small electric bulb fixed on the wall of a room is to be obtained on the opposite wall 3 m
away by means of a large convex lens. What is the maximum focal length of the lens required for the
purpose? [0.75 m]

Solution
Object distance = −u, Image distance = (3 − u)
1 1 1
By Lens formula, − =
v u f
1 1 1 1 1 1 u +3− u 1
− = or + = or =
(3 − u) −u f (3 − u) u f u(3 − u) f
3f = 3u − u2  u2 − 3u + 3f = 0
+3 ± 9 − 2f
u=
2
For the image to be real, 9 − 12f ≥ 0 or 9 ≥ 12f or f ≤ 0.75 m
Thus, the maximum focal length of the lens required is 0.75 m.
34. Calculate the focal length of the thin lens shown in the figure. The points
C1 and C2 represent the centres of curvature. Radii of curvature are 10 cm and
20 cm. (Refractive index of the material of the lens = 1.5) [40 cm] C1 C2

Solution
From the figure, it is seen that both the radii of curvature are positive. Thus, R1 = + 10 cm and
R2 = + 20 cm. The focal length is given by
1  1 1 
= (n − 1)  − 
f  R1 R 2 
C1 C2
1  1 1 
= (1.5 − 1)  −  II-face
f  10 cm 20 cm 
I-face
1 1
= 0.5 × = or f = 40 cm
20 cm 40 cm

Long answer questions (5M)


35. Obtain the relation between u, v, n and R when a spherical surface is concave towards the denser
medium and a point object is in the rarer medium, forming real image.
Solution
Refer Module 7A
36. Derive lens maker’s formula. [June 2015, June 2016]
Solution
Refer Module 7A
37. Obtain lens formula.
Solution
Refer Module 7A

27
2PBBBCPM7AS

38. Derive the expression for the equivalent focal length of two thin lenses in contact. Write the expression
for the power of the combination. [March 2016]
Solution
Refer Module 7A

Classwork Problems
3 4
39. A spherical surface of radius of curvature 1 m separates two media of refractive indices and . The
2 3
surface is concave towards the rarer medium. An object is kept in the first medium at a distance of 0.6 m
from the surface. Calculate the distance of the image. [–0.5 m]
Solution
u = PO = −0.6 m
R = PC = + 1 m
3 4
n1 = , n 2 = , v = ?
2 3 n = 4/3
n = 1.5
1 2
n 2 n1 (n 2 − n1 ) (Denser) (Rarer)
− =
v u R O P C
4 3  4 3 
 − 
3 − 2 = 3 2
v (−0.6) +1
4 1 1 4 1 1
+ =−  =− −
3v 0.4 6 3v 6 0.4
4 −0.4 − 6 6.4 4 × 2.4
= =−  v=− = −0.5m
3v 2.4 2.4 3 × 6.4
40. An object of size 3.0 cm is placed 14 cm in front of a concave lens of focal length 21 cm. Describe the
features of the image produced by the lens. [Virtual, erect, at 8.4 cm, size decreases] [June 2019]
Solution
ho = 3 cm, u = −14 cm, f = −21 cm, v = ?, hI = ?
1 1 1
By lens formula, − =
v u f Object
1 1 1 O
− = F F
v (−14) −21 u
f
1 1 1 −2 − 3 5
=− − = =−
v 21 14 42 42
 v = − 8.4 cm
v h
Magnification, m = = I
u hO
−8.4 h I
=  hI = 1.8 cm
−14 3
hI is positive,  the image formed is virtual and erect at a distance of 8.4 cm from the lens. Height of the
image is 1.8 cm. The object and the image are on the same side of the lens.

28
2PBBBCPM7AS

41. (i) If f = 0.5 m for a glass lens, what is the power of the lens?
(ii) The radii of curvature of the faces of a double convex lens are 10 cm and 15 cm. Its focal length is
12 cm. What is the refractive index of glass?
(iii) A convex lens has 20 cm focal length in air.
(a) What is the focal length when immersed in water?
(b) When immersed in a liquid of n = 1.6?
(Refractive index of air-water = 1.33, refractive index for air-glass = 1.5.)
(iv) The radii of curvature of two surfaces of a convex lens are 0.2 m and 0.22 m. Find the focal length
of the lens if refractive index of the material of the lens is 1.5. Also, find the change in the focal
length, if it is immersed in water of refractive index 1.33
[(i) 2 D (ii) 1.5 (iii) 78.2 cm, -160 cm (iv) 4.4 m, 13.2 m][June 2018]
Solution
(i) Power = +2 dioptre.
(ii) Here, we have f = +12 cm, R1 = +10 cm, R2 = –15 cm.
Refractive index of air is taken as unity.
1  1 1 
We use the lens formula of = (n − 1)  −  . The sign convention has to be applied for f,
f  R1 R 2 
1  1 1 
R1 and R2. Substituting the values, we have = (n − 1)  −  . This gives n = 1.5.
12  10 −15 
(iii) (a) For a glass lens in air, n2 = 1.5, n1 = 1, f = +20 cm.
1 1 1 
Hence, the lens formula gives = 0.5  − 
20  R1 R 2 
For the same glass lens in water, n2 = 1.5, n1 = 1.33.
1.33 1 1 
Therefore, = (1.5 − 1.33)  − 
f  R1 R 2 
Combining these two equations, we find f = + 78.2 cm
(b) For a glass lens in air, n2 = 1.5, n1 = 1, f = +20 cm.
1 1 1 
Hence, the lens formula gives = 0.5  − 
20  R1 R 2 
For the same glass lens in liquid, n2 = 1.5, n1 = 1.6.
1  1.5   1 1 
Therefore, =  − 1  − 
f  1.6   R1 R 2 
Combining these two equations, we get, f = -160 cm.
(iv) Here, we have ng = 1.5, R1 = 0.2 m, R2 = –0.22 m, f = ?
1  1 1 
Using lens maker’s formula of = (n − 1)  − 
f  R1 R 2 
1  1 1 
= (1.5 − 1)  + 
f  0.2 0.22 
1  0.22 − 0.2 
= (0.5)   = 4.772
f  0.044 
f = 0.21 m

29
2PBBBCPM7AS

When immersed in water,


 
 n −1 
fw  g   f w = 4f air = 4 × 0.21 m = 0.84m
=
f air  n g 
 −1
 nw 
Change in focal length = 0.84 – 0.21 = 0.63 m
42. What is the focal length of a convex lens of focal length 30 cm in contact with a concave lens of focal
length 20 cm? Is the system a converging or a diverging lens? Ignore thickness of the lenses.
[diverging, 60 cm]
Solution
Given, focal length of the convex lens, f1 = 30 cm,
Focal length of the concave lens, f2 = −20 cm
Using the formula of combination of lenses, we get
1 1 1 1 1 2−3 1
= + = − = =−
f eq f1 f 2 30 20 60 60
feq = −60 cm
Since, the focal length of combination is negative, the combination behaves as a diverging lens.
43. Two lenses of focal lengths 0.20 m and 0.30 m are kept in contact. Find the focal length of the
combination. Calculate powers of the lenses and of the combination. [5 D, 3.3 D, 8.3 D] [March 2014]
Solution
f1 = 0.20 m, f2 = 0.30 m, feq = ?, P1 = ?, P2 = ?, Peq = ?
1 1 1
= +
f eq f1 f 2
1 1 1
= + = 5.0 + 3.3 = 8.3  Peq = 8.3 D
f eq 0.20 0.30
1
f eq = = 0.12 m
8.3
1
P (in dioptre) =
f (in metre)
1 1
P1 = = 5.0 D , P2 = = 3.3 D
0.20 0.30
Peq = P1 + P2 = 5.0 + 3.3 = 8.3 D

Practice Exercise
44. A small air bubble within a glass sphere of radius 20 mm and n = 1.54 appears to be 10 mm below the
surface, when looked along the diameter containing the bubble. Find the real depth of the bubble along
the line of sight. [12.13 mm]
Solution
Object distance, P1O = u = ?
Image distance, P1I = v = −10 mm
Radius of curvature of AP1B = P1C = R = −20 mm

30
2PBBBCPM7AS

n1 = ng = 1.54, n2 = na = 1
n n (n − n1 )
Refraction formula, 2 − 1 = 2 n2 = n a
v u R n1 = ng
A
1 1.54 (1 − 1.54)
− = P1 I
−10 u −20 O C
B
1 1.54 0.54
− =
−10 u 20
1.54 1 0.54
=− −
u 10 20
1.54 −2 − 0.54 2.54
= =−
u 20 20
20 × 1.54
u=−
2.54
P1O = u = −12.13 mm. Negative sign is in accordance with sign convention.
45. A double convex lens of focal length 0.444 m has radii of curvature of 0.4 m and 0.5 m respectively.
Calculate the refractive index of the material of the lens. [1.5]
Solution
f = 0.444 m, R1 = 0.4 m R2 = − 0.5 m, n = ?
1  1 1 
= (n − 1)  − 
f  R1 R 2 
1  1. 1 
= (n − 1)  − 
0.444  0.4 (−0.5) 
 1 1   0.5 + 0.4 
= (n − 1)  +  = (n − 1)  
 0.4 0.5   0.2 
1  0.9 
= (n − 1)  
0.444  0.2 
0.2
(n − 1) =  n = 1.50
0.444 × 0.9
46. An equiconvex lens of focal length 0.1 m has an object at a distance of 0.4 m from it. If the lens is sliced
into two identical planoconvex lenses and one of them retained in the original position of the lens,
calculate the distance through which the screen has to be moved and the direction of motion to get the
image on the screen. (Refractive index of the material of the lens = 1.5) [0.267 m]
Solution
• For the equiconvex lens, u1 = − 0.4 m, n = 1.5
1 2
= (n − 1)
f1 R
R = (n −1) × 2 × f1 = (1.5 −1) × 2 × 0.1 = 0.5 × 2 × 0.1 = 0.10 m
1 1 1 1 1
Lens formula − =  = 10 −
v1 u1 f1 v1 0.4
v1 = 0.133 m

31
2PBBBCPM7AS

1 1 0.5 × 1 1
• For a planoconvex lens, = (n − 1) =  =5
f2 R 0.1 f2
 f2 = 0.2 m
1 1 1 1 1
= + = + = 2.5
v 2 f 2 u 2 0.2 (−0.4)
1
 v2 = = 0.4
2.5
∴ ∆ v = v 2 − v 1 = 0.4 − 0.133 = 0.267 m
47. Wavelength of monochromatic light on passing from air to glass reduces to (2/3) times its value in air.
An object held at 1.25 m from an equiconvex lens made of the above glass has its image at a distance of
5 m from the lens. Calculate the radius of curvature of the lens surface. [1 m]
Solution
ng λa
nga = =
n a λg
λa 3
 a ng = =
2 λ 2
3 a
1 1 1 1 1 1
− =  − =  f =1 m
v u f 5 (−1.25) f
1 2
= (n − 1)  
f R
 2   3  2 
 R = (n − 1)   =  − 1  = 1 m.
 f   2  1 
48. Double-convex lenses are to be manufactured from a glass of refractive index 1.55, with both faces of
the same radius of curvature. What is the radius of curvature required if the focal length is to be 20 cm?
[22 cm]
Solution
ng = 1.55, R1 = +R, R2 = −R, f = 20 cm
1  1 1 
By lens maker’s formula, = (1 n 2 − 1)  − 
f  R1 R 2 
Unless otherwise mentioned, the lens is assumed to be in air.
n ng
Thus, 1 n 2 = 2 = = 1.55
n1 n air
1 1 1  0.55 × 2
= (1.55 − 1)  − =
20  R (−R)  R
R = 20 × 0.55 × 2 = 22 cm  R = 22 × 10−2 m
49. Two equi-convex lenses of refractive index 1.5 have radii of curvature 0.15 m and 0.10 m respectively.
They are placed in contact with their curved surfaces facing each other. The space between them is filled
with oil of refractive index 1.6. Calculate the focal length of the combination. [0.15 m]
Solution
1 1 1 
Since, = (n − 1)  − ,
f  R1 R 2 

32
2PBBBCPM7AS

1  1 1  1
= (1.5 − 1)  − =
f1  0.15 −(0.15)  0.15
1  1 1  1
= (1.5 − 1)  − = º + +
f2  (0.1) (0.1)  0.1
1 1  1 1 
= = (1.6 − 1)  − 
f liq f3  (−0.15) (+0.1)  L1 Lliq L2 L1 Lliq L2
Fig.
−0.6 × 250
= = −0.2 × 50 = −10 For lens L1: |R1| = |R2| = 0.15m
15 For lens L2: |R1| = |R2| = 0.1 m
1 1 1 1 1 1 1 For liquid lens: (R1)liq = −0.15 m
= + + = + − 10 =
f eq f1 f 2 f 3 0.15 0.1 0.15 (R2)liq = −0.1 m

∴ feq = 0.15 m.
50. A convex and concave lens of focal lengths 10 and 20 cm are kept coaxially in contact. A point object is
placed at a distance of 50 cm from the combination. Find the position of the final image due to the
combination [assume that the lenses are of negligible thickness] [33.33 cm]
Solution
1 1 1 1 1 1
= + but = −
f eq f1 f 2 f eq v u
1 1 1 1
 − = +
v u f1 f 2
1 1 1 1
− = −
v −50 10 20
 v = 33.33 cm

9.5 Optical Instruments


Exercise
Very short answer questions (VSA)
1. What is the nature of the image formed in a simple microscope?
Solution
The image formed is virtual, erect and magnified
2. Why do we prefer a magnifying glass of smaller focal length?
Solution
The magnifying power will be large when focal length (f) of the magnifying glass is small.
  D 
Magnifying power, m =  1 +  
  f 
3. Calculate the magnifying power of a magnifying glass of 5 cm focal length. Distance of distinct vision
25 cm.
Solution
f = 5 cm, D = 25 cm.
D 25
The magnifying power (m) of a magnifying glass is given by m = 1 + = 1 + =1+ 5 = 6 .
f 5

33
2PBBBCPM7AS

4. What is the nature of the final image formed in a compound microscope?


Solution
In a compound microscope, the final image is inverted w.r.t the object. It is virtual and magnified.
5. What is optical tube length of a compound microscope?
Solution
The distance between the second focal point of the distance (Fo on the right hand side) and the first focal
point of the eyepiece (Fe on the left hand side) is called optical tube length of a compound microscope.
(It is nearly equal to vo).
6. On what factors does the magnifying power of compound microscope depend?
Solution
The magnifying power of a compound microscope depends on
(i) focal length fo of the objective lens (ii) focal length fe of the eye lens.
7. If half of the objective of a telescope or of a microscope is painted black will there be any change in the
magnification produced by the telescope?
Solution
There will be no change in the magnification but the intensity of the image will be reduced.
8. What do you mean by normal adjustment of a telescope?
Solution
When the final image is formed at infinity, the telescope is said to be in normal adjustment.
9. What is the distance between the objective and eyepiece of an astronomical telescope in normal
adjustment?
Solution
Length of telescope in normal adjustment = fo + fe.

10. What is the nature of the final image formed in an astronomical telescope?
Solution
The final image formed is virtual, magnified and inverted w.r.t the object.
11. An astronomical telescope set for normal adjustment has a magnifying power 20. If the focal length of
the objective is 2 m, what is the focal length of the eyepiece?
Solution
f f 2
As m = o , f e = o = = 0.1 m
fe m 20
12. A reflecting type telescope has, a concave reflector of curvature 1 m. Calculate the focal length of the
eyepiece to secure a magnification of 10.
Solution
R 1 f f 0.5
f o = = = 0.5 m As m = o , f e = o = = 0.05 m
2 2 fe m 10

34
2PBBBCPM7AS

Short answer questions (SA - 2 M)


13. What is the magnifying power of a simple microscope when the image is formed (i) at least distance (D)
of distinct vision (D) (ii) at infinity?
Solution
D D
(i) m = 1 + (ii) m =
f f
14. At what position of the image in a simple microscope the magnifying power is (i) maximum
(ii) minimum?
Solution
(i) When the image is formed at the least distance of distinct vision.
(ii) When the image is formed at infinity.
15. The near vision of an average person is 25 cm. To get an angular magnification of 10, what should be the
power of a simple microscope?
Solution
The near vision of an average person is 25 cm.
D
To get an angular magnification of 10, m =
f
D 25 1
f= = = 2.5 cm = 0.025 m P= = 40 dioptre
m 10 0.025
16. Why should the focal length of the eye-piece of a microscope be small?
Solution
The focal length of the eye-piece of a microscope should be small to get greater magnification, as seen
v  D
from the formula m = o  1 +  .
u o  fe 

17. A small telescope has an objective lens of focal length 144 cm and an eye-piece of focal length 6.0 cm.
What is the magnifying power of the telescope? What is the separation between the objective and the
eye-piece? [−24, 150 cm]
Solution
Focal length of objective lens, fo = 144 cm
Foal length of eye-piece, fe = 6 cm
Magnifying power of the telescope in normal adjustment, (i.e. when the final image is formed at infinity)
f 144
m=− o =− = −24
fe 6
∴ Separation between lenses L = fo + fe = 144 + 6 = 150 cm
18. Why the objective of a telescope should have large focal length and have large aperture?
Solution
 f 
The focal length should be of large for large magnification  m = o  and the aperture should be large so
 fe 
that the objective collects large amount of light from distant objects so that image is bright.
Also resolving power increases (Refer Ch 10.2, 7A)

35
2PBBBCPM7AS

Short answer type questions (SA - 3M)


19. Explain the working of a simple microscope and obtain the expression for magnifying power, when the
image is at distance of distinct vision.
Solution
A simple microscope is a convex lens of short focal length. It is also called a magnifying glass.
The object is kept near the lens at a
A’
convenient distance, less than the focal
length of the lens. The eye is held close to
the lens. A virtual, erect and magnified A
image is formed. Figure shows the ray F b F
diagram of a simple microscope. A small B’ B O
u
D
object AB is placed within the principle
A
focus of the lens at a distance u from it. Its
distance is adjusted such that a virtual, erect, a
B O
enlarged image is formed at the least D
distance of distinct vision D.
Simple microscope – Image at D
β is the angle subtended at O the optic
centre, by the image formed at distance D from lens.
α = angle subtended by the object at O, when placed at a distance D from lens
β is larger than α.
The magnifying power or angular magnification (m) of a microscope is defined as the ratio of the angle
subtended by the image at the eye (β) to the angle subtended by the object at the unaided eye (α), when
both are placed at the least distance of distinct vision.
angle β subtended by the image as seen through the instrument β
Thus, m = =
angle α subtended by the object kept at near point α
For small values of β and α we can write
tan β
m=
tan α
AB / u
∴ m=
AB / D
D
i.e., m = …(1)
u
1 1 1
We have for a convex lens = − …(2)
f v u
By sign convention, both v and u are negative.
1 1 1
∴ = −
f − v −u
Multiplying this equation throughout by v we get,
v v v v v v v
= −  = −1 +  1 + =
f − v −u f u f u
D D
But v=D ∴1 + =
f u
D  D
Since m =  m = 1 +  …(3)
u  f 

36
2PBBBCPM7AS

20. Draw a labelled ray diagram of a compound microscope. Write the expression for magnifying power.
[March 2015]
Solution

 L  D 
m = m o m e =  1 + 
 f o  f e 
21. Draw a labelled diagram of an astronomical telescope. Write the expression for magnifying power.
Solution

fo
m=
fe

22. Draw a labeled diagram of Cassegrain telescope. Write the expression for magnifying power.
Solution
Objective mirror
Secondary
mirror

Fo F
Eyepiece

Cassegrain reflecting telescope


fo
M=
fe

37
2PBBBCPM7AS

Long answer questions (5M)


23. With a neat diagram, describe the principle, construction and working of a compound microscope.
Solution
Refer Module 7A
24. Derive an expression for the magnifying power of a compound microscope when final image is formed
(i) at near point (ii) at infinity.
Solution
Refer Module 7A
25. Describe the construction and working of an astronomical telescope.
Solution
Refer Module 7A
26. Derive an expression for the magnifying power of an astronomical telescope when final image is formed
at infinity.
Solution
Refer Module 7A

Classwork Problems
27. A man with normal near point (25 cm) reads a book with small print using a magnifying glass: a thin
convex lens of focal length 5 cm.
(a) What is the closest and the farthest distance at which he should keep the lens from the page so that
he can read the book when viewing through the magnifying glass?
(b) What is the maximum and the minimum angular magnification (magnifying power) possible using
the above simple microscope? [(a) −4.2 cm and −5 cm, (b) 6 and 5]
Solution
Near point, u = 25 cm
(a) Focal length f = + 5 cm, v = − 25 cm
1 1 1
By the lens formula, = −
f v u
1 1 1 1 1 1 −1 − 5 6
= −  =− − = =−
5 −25 u u 25 5 25 25
u = − 4.2 cm
For the farthest distance v′= ∞ and f = 5 cm
1 1 1 1 1 1
= −  = −
f v′ u ′ 5 ∞ u′
u′ = − 5 cm
Thus, the closest and the farthest distances are (−) 4.2 cm and (−) 5 cm, respectively.
D 25
(b) Maximum angular magnification m max = = =6
u 25
6
D 25
Minimum angular magnification is at the farthest distance. m min = = =5
u′ 5

38
2PBBBCPM7AS

28. A person with a normal near point (25 cm) using a compound microscope with objective of focal length
8.0 mm and an eye-piece of focal length 2.5 cm can bring an object placed at 9.0 mm from the objective
in sharp focus. What is the separation between the two lenses? Calculate the magnifying power of the
microscope. [9.47 cm, −88]
Solution
• Given, focal length of objective fo = 8 mm = 0.8 cm
Focal length of eye-piece fe = 2.5 cm
Distance of object from objective, uo = − 9 mm = − 0.9 cm
Distance of image from eye-piece, ve = −25 cm
Using lens formula for the eye-piece, we get
1 1 1
− =
ve u e f e
1 1 1 1 1 −1 − 10 11
= − = − = =−
u e v e f e (−25) 2.5 25 25
ue = −2.27 cm …(1)
• Using lens formula for the objective lens, we get
1 1 1
− =
vo u o fo
1 1 1 1 1 0.9 − 0.8 0.1
= + = − = =
v o f o u o 0.8 0.9 0.72 0.72
Distance of the image from the objective lens,
vo = 7.2 cm …(2)
• Separation between two lenses, (Using Eq. (1) and (2))
L = | ue | + |vo| = 2.27 + 7.2
L = 9.47 cm
v  D 7.2  25 
• Magnifying power of the compound microscope, m = o  1 +  = 1 + 
u o  f e  −0.9  2.5 
m = – 88 (Final image is inverted w.r.t. the object)
29. A small telescope has an objective of focal length 140 cm and an eye-piece of focal length 5.0 cm.
(a) Calculate the magnifying power. [24]
(b) What is the separation between the objective lens and the eye-piece? [145 cm]
(c) If this telescope is used to view a 100 m tall tower 3 km away, what is the height of the image of the
tower formed by the objective lens? [4.7 cm]
Solution
Given, fo = 140 cm fe = 5 cm and d = 25 cm
f 140
(a) m = o = = 24
fe 5
(b) In normal adjustment, the separation between eye-piece and objective = fo + fe = 140 + 5 = 145 cm
(c) Height of the tower, hO = 100 m
Distance of tower, u = 3 km = 3000 m h0
θ0
h O 100 1 θi
Angle subtended by the object, θ0 = = = rad u hx
u 3000 30
v
Angle subtended by the image.

39
2PBBBCPM7AS

hI h
θI = = I (∵ hI = height of image tower)
f o 140
As θ0 = θ1
1 h
= I
30 140
14
h I = = 4.7 cm
3
Thus, the height of the image of the tower is 4.7 cm.

Practice Exercise
30. An angular magnification (magnifying power) of 30 X is desired using an objective of focal length
1.25 cm and an eye-piece of focal length 5 cm. How will you set up the compound microscope?
[|u| = 1.5 cm, L = 11.67 cm]
Solution
Focal length of objective, fo = 1.25 cm
Focal length of eye-piece, fe = 5 cm
Least distance of distinct vision, d = 25 cm
Magnifying power = Angular magnification mc = 30
The magnification produced by eye-piece
D 25
m =1+ =1+ =6
fe 5
The magnification produced by the microscope,
m = mo × me, where, mo is the magnification produced by the objective lens.
30 = mo × 6
∴ mo = 5
Magnification due to the objective lens,
v
mo = o
uo
−v
5= o
uo
vo = − 5uo
1 1 1
• Using Lens formula for objective lens, = −
fo vo u o
1 1 1 6
= − =− ...(1)
1.25 −5u o u o 5u o
6
From Eq. (1) we get u o = − × 1.25 = −1.5 cm
5
v o = − 5uo = − 5 (−1.5) = 7.5 cm
Thus, the object should be placed at a distance of 1.5 cm from the objective lens to get the desired
magnification.
1 1 1
• By lens formula for eye-piece, = −
f e ve u e
1 1 1 1 1 6
= − =− − =− (∵ ve = −25 cm)
u e ve f e 25 5 25

40
2PBBBCPM7AS

ue = − 4.17 cm
∴ The separation between the objective and the eye-piece
| v o | + | ue | = 4.17 + 7.5 = 11.67 cm
31. A Cassegrain telescope uses two mirrors as shown in figure. Such a telescope is built with the mirrors
20 mm apart. If the radius of curvature of the large mirror is 220 mm and the small mirror is 140 mm,
where will the final image of an object at infinity be? [0.315 m]
Objective
mirror

I Secondary
Eye piece
mirror

Solution
Objective
mirror

I Secondary
Eye piece
mirror

90 mm

110 mm 20 mm

Distance between the objective mirror and the secondary mirror, d = 20 mm


Radius of curvature of the objective mirror = |R1| = 220 mm
220
∴ Focal length of the objective mirror, f1 = = 110 mm
2
Radius of curvature of the small mirror = |R2| = 140 mm
140
∴ Focal length of the small mirror, f 2 = = 70 mm
2
The image formed by the objective mirror will serve as an object for the secondary mirror.
For the secondary mirror,
u = f1 − d = 110 − 20 = 90 mm, f 2 = 70 mm,
1 1 1
By mirror formula, + =
v u f2
u = + 90 mm, f2 = +70 mm, v = ?
1 1 1
+ =
v (+90) (+70)
1 1 1 90 − 70 20
= − = =
v 70 90 70 × 90 70 × 90
70 × 90
v= = 315 mm = 0.315 m
20

41
10. Wave Optics
Huygens’ Wave Theory, Doppler Effect, Interference
Exercise
Very short answer questions (VSA)
1. What is a wavefront? [March 2014]
Solution
Locus of points in a medium with a certain (constant) phase is called a wavefront.
2. What is the phase difference between two diametrically opposite points on a spherical wavefront?
Solution
Zero. Phase is constant at all points on a wavefront.
3. How to get a cylindrical wavefront?
Solution
A linear source of light gives rise to a cylindrical wavefront.
4. What is the shape of the wavefront in the following cases?
(a) Light diverging from a point source.
(b) Light emerging out of a convex lens, when a point source is placed at its focus.
(c) A portion of the wavefront of light from a distant star intercepted by the earth.
Solution
(a) Spherical. (b) Plane. (c) Plane.
5. Write the expression for Doppler shift in light.
Solution
When there is a relative motion between the source and the observer the Doppler shift is given by
∆λ v
= − radial , (v radial << c)
λ c
6. Two stars are moving along a direction away from the earth with the same speed v. What is the change
in wavelength of light from one star, detected at the other star?
Solution
v
Since there is no relative velocity v between the source and observer (i.e. two stars), ∆λ = λ = 0 and
c
hence there is no Doppler shift.
7. State the principle of superposition of waves?
Solution
At a particular point in an optical medium, the resultant displacement produced by a number of waves of
same nature is the vector sum of the displacements produced by the individual waves.
8. What is interference of light?
Solution
Interference of light is the phenomenon of modification in the intensity of light due to redistribution of
light energy in the region of superposition of two or more light waves.
9. Is interference exhibited by any type of wave?
Solution
Yes. (Longitudinal or transverse, mechanical or electromagnetic, matter waves)

42
2PBBBCPM7AS

10. What is constructive interference?


Solution
Constructive interference is said to take place at a point if two light waves arrive at that point in phase
leading to the formation of a bright point.
11. What is destructive interference?
Solution
Destructive interference is said to take place at a point if two light waves arrive at that point out of phase
leading to the formation of a dark point.
12. State the condition for constructive interference in terms of (i) phase difference (ii) path difference of
two waves.
Solution
(i) The phase difference between the two waves must be an integral multiple of 2π or an even multiple
of π i.e. φ = 2nπ, where n = 0, 1, 2, …
(ii) The path difference between two waves must be an integral multiple of λ i.e. δ = nλ, where
n = 0, 1, 2, 3, …
13. State the condition for destructive interference in terms of (i) phase difference (ii) path difference of two
waves.
Solution
(i) The phase difference between the two waves must be an odd multiple of π i.e. φ = (2n − 1)π, where
n =1, 2, 3, …
λ
(ii) The path difference between two waves must be an odd multiple of λ/2. i.e. δ = (2n − 1) where
2
n = 1, 2, 3, …
14. What is fringewidth?
Solution
Fringewidth is the distance between two consecutive bright or dark fringes.
15. Write the expression for fringewidth in Young’s double slit experiment.
Solution
β = λD/d, λ = wavelength of light, D = distance between the coherent sources and the screen,
d = distance between the coherent sources.
16. What are coherent sources?
Solution
The sources are said to be coherent, if they emit light waves of same frequency with constant phase
difference.
17. How does the intensity vary in an interference pattern?
Solution
In the interference pattern, intensity of light is maximum at the points of constructive interference and
minimum at the points of destructive interference.
18. Why is the central fringe always bright in the double slit experiment?
Solution
The waves arriving at the point of central fringe will be in phase and constructive interference takes
place.

43
2PBBBCPM7AS

19. What happens to the fringewidth if (i) yellow light is replaced by blue light and (ii) blue light is replaced
by red light?
Solution
(i) Decreases
(ii) increases. (β ∝ λ, λR > λY > λB)
20. What will be the fringe pattern if white light is used instead of monochromatic light?
Solution
The central fringe will be bright and white, where as each one of the other bright fringes will be
coloured.
21. What happens if one of the slits is covered in the double slit experiment?
Solution
Fringe pattern due to interference disappears. However, diffraction pattern (Fresnel type) may be seen.
22. In Young’s double slit experiment, what happens to the fringewidth, when
(i) the distance between slits and the screen is doubled?
(ii) the distance between the slits is doubled?
(iii) the width of the slits is increased, keeping the distance between them same?
(iv) the experimental arrangement (slits and screen) is immersed in water?
Solution
(i) β ∝ D. Therefore, the fringewidth will be doubled.
1
(ii) β ∝ . Therefore, the fringewidth will be halved.
d
(iii) The contrast between bright and dark fringes decreases.
(iv) When the experimental arrangement is immersed in water, the fringewidth decreases.
 βair λ 
 β water = , sin ce λ water = air 
 n n 
23. Write the expression for (i) linear distance of nth bright fringe from the centre (ii) angular position of nth
bright fringe in double slit experiment.
Solution
nλ D
(i) Linear distance of nth bright fringe from the centre, x n = nβ = , where n = 0, 1, 2, 3.......
d

(ii) Angular Position of the nth bright fringe is given by sin θ = , where λ is wavelength of light, D is
d
the distance between the slits and the screen, d is the distance between the slits.
24. Write the expression for (i) distance of dark fringe of nth order from the centre (ii) angular position of
dark fringe of order n in Young’s experiment.
Solution
(2n − 1)β (2n − 1)λD
(i) Distance of nth dark fringe from the centre, x n = = , where n = 1, 2, 3 …
2 2d
λ
(ii) Angular position of nth dark fringe, sin θ = (2n − 1) , where n = 1, 2, 3 …
2d

44
2PBBBCPM7AS

Short answer questions (SA - 2 M)


25. Draw a schematic diagram showing the propagation of a (i) spherical wavefront (ii) x.
Solution

S S
S S

(a) (b)
(a) spherical wavefront (b) wavefront (c)
plane (c)
26. How does Doppler effect in light differs from that in sound?
Solution
Change in wavelength of light due to the relative motion between the source and observer does not
depend on whether (i) the source is at rest and observer is in motion, or (ii) the observer is at rest and
source is in motion. But in sound, change in wavelength differs in the cases, (i) and (ii).
27. The intensities of two interfering beams in a Young’s experiment are in the ratio 9:1. Compare the
intensities of the bright and dark fringes. [4 : 1]
Solution
2
I1 9 a1 9 3 Ibright ( a1 + a 2 ) (3 + 1) 2 16 4
= ∴ = = (∵I∝a ) 2
= 2
= = = .
I2 1 a2 1 1 Idark ( a1 − a 2 ) (3 − 1)2 4 1
28. Calculate the path difference corresponding to a phase difference of [π/10] radians for two waves of
wavelength 500 nm. [25 mm]
Solution
Phase difference of 2π radian = path difference of λ
π λ π λ
∴ Phase difference of   radian = path difference of × =
 
10 2 π 10 20
500 nm
Hence, path difference = = 25nm.
20
29. Two waves of wavelength 500 nm have a path difference of 1000 nm. What is the phase difference
between the two waves? [4π radian]
Solution
2π 2π
Phase difference × path difference = × 1000 nm = 4π radian.
λ 500 nm
30. Distinguish between constructive and destructive interference.
Solution
Constructive interference Destructive interference
(a) Constructive interference results in a bright (a) Destructive interference results in a dark point.
point.
(b) Condition: (b) Condition:
Path difference = nλ, n = 0, 1, 2 … Path difference = (2n − 1) λ/2, n = 1, 2, 3 …

45
2PBBBCPM7AS

31. Can you observe sustained interference pattern due to the two head lights of a car? Justify.
Solution
No. The two sources are not coherent.
32. What are coherent sources? Describe a simple method to observe interference fringes.
Solution

Two sources are said to be coherent, if they emit light waves of same frequency with constant phase
difference.
33. Draw the intensity distribution graph of YDSE interference pattern
Solution
I

2
I0 µ 4a

Df
-7p -5p -3p -p p 3p 5p 7p
Fig. Variation of intensity with phase difference

34. In Young’s double slit experiment, light of wavelength 600 nm is used. Find the phase difference and
path difference for the 6th dark fringe. [3.3 µm]
Solution
Phase difference for 6th dark fringe = φ = (2n − 1)π = (2 × 6 − 1)π = 11π (for 6th dark fringe take n = 6).
λ λ 11λ 11 × 600
Path difference = δ = (2n − 1) = (2 × 6 − 1) = = nm = 3300 nm = 3.3 µm
2 2 2 2
35. Two narrow parallel slits 0.5 mm apart are illuminated with monochromatic light of wavelength
4500 Å. Calculate the fringewidth obtained on a screen 2 m away from the slit. [1.8 mm]
Solution
Dλ 2 × 4500 × 10−10
β= = −3
= 1.8 × 10−3 m
d 0.5 × 10
36. Amplitudes of the interfering waves in a Young’s experiment are in the ratio 1: 2. Compare the
intensities of the bright and dark fringes. [9 : 1]
Solution
Imax (a1 + a 2 ) 2 (2 + 1)2 9
= = =
Imin (a1 − a 2 )2 (2 − 1) 2 1

46
2PBBBCPM7AS

37. Obtain the condition for constructive interference in terms of path difference in a Young’s double slit
experiment. [March 2015, June 2016]
Solution
φ
Intensity is maximum when cos 2   = 1  φ = 0, ± 2π, ± 4π, …, Imax = 4I0
2
Condition for maxima in terms of phase difference φ = ±2nπ, ± 2nπ, where n = 0, 1, 2,
This corresponds to bright points on the screen. The bright points on the screen are called interference
maxima. Interference at such points is called constructive interference
Condition for maximum in terms of path difference, S1P1 ~ S2P1 = nλ where n = 0, 1, 2,
38. Obtain condition for destructive interference in terms of path difference in a Young’s double slit
experiment.
Solution
φ
Intensity is minimum when cos 2   = 0  φ = ± π, ± 3π, ± 5π, …,
2
Imin = 0
Condition for minima in terms of phase difference, φ = (2n + 1)π where n = 0, 1, 2, 3,
This corresponds to dark points on the screen. Interference at such points is called destructive
interference.
λ
Condition for minima in terms of path difference, S1P2 ~ S2 P2 = (2n + 1) where, n = 0, 1, 2, 3,
2
39. What will be the nature of the fringes if the slits in an interference experiment are illuminated by white
light?
Solution
When the slits are illuminated with white light:
The central bright fringe appears white.
Each of the other bright fringes consists of coloured parallel strips. .
40. The intensity of two waves are I and 4I respectively. What is the resultant intensity at a point where the
two waves arrive with a phase difference of (i) (π / 4) rad (ii) π rad [(i) 7I (ii) I]
Solution
Ires = I1 + I2 + 2 I1I 2 cos φ
π 1 1
(i) cos = . Ires = I + 4I + 2 I × 4I × = 7I
3 2 2
(ii) cos π = −1. Ires = I + 4I + 2 I × 4I(−1) = I
41. Calculate the percentage change in the fringewidth in a Young’s double slit experiment when the
distance between the double slit and screen is increased by 20% while the slit separation is increased by
50%. [20%]
Solution
 20 
D′ = D +   D = 1.2 D
 100 
 50 
d′ = d +   D = 1.5d
 100 

47
2PBBBCPM7AS

Dλ β′ ( D′ / d′ ) 1.2 4
Since β = = = = = 0.8
d β ( D / d ) 1.5 5
β′
1− = 1 − 0.8 = 0.2
β
∆β
= 0.2  % change = 20%
β
42. In a Young’s double-slit experiment, the slits are separated by 0.28 mm and the screen is placed 1.4 m
away. The distance between the central bright fringe and the fourth bright fringe is measured to be
1.2 cm. Determine the wavelength of light used in the experiment. [600 nm]
Solution
Given: d = 0.28 mm, D = 1.4 m, x4B = 1.2 cm, λ = ?
Distance of a bright fringe from the central bright fringe is given by
nλ D
xn =
d
x .d 1.2 × 10−2 × 0.28 × 10−3
∴ λ= n = = 6 × 10−7 = 600 nm
nD 4 × 1.4
43. Light of wavelength 5000 Å falls on a plane reflecting surface. (a) What are the wavelength and
frequency of the reflected light? (a) For what angle of incidence is the reflected ray normal to the
incident ray? [(a) 5000 Å, 6 × 1014 Hz (b) 45°]
Solution
(a) Wavelength of reflected light = 5000 Å
c 3 × 108
Frequency of reflected light is, f = = −7
= 6 × 1014 Hz
λ 0 5 × 10
(b) Angle of incidence = angle of reflection
If the reflected ray has to be normal to the incident ray, i = 45°.

Short answer type questions (SA - 3 M)


44. Explain from Huygens’ theory, how does light from a point source spreads in a medium?
Solution
Let us assume that we have a point source (O) of light located in a homogeneous isotropic medium.
Light energy liberated from the point source travels in all directions in the optical medium at a constant
speed v and reaches every point equidistant from the source at the same time. This distance is given by
x = v∆t, where ∆t is the time elapsed since the optical disturbance left the source
O and v is the propagation velocity of the disturbance. The locus of all these
points is a spherical surface. Every point in this surface is at the same phase and
S
such a surface is called a wavefront. Thus, a wavefront is a surface in an optical
medium in which electromagnetic disturbance is propagating, such that all points
on the surface are at the same phase.
For a point source, the wavefront is spherical. (a)
Spherical wavefronts

48
2PBBBCPM7AS

45. Explain Young’s double slit experiment. [June 2015]


Solution
Young (1802) performed his famous light
P Y
interference experiment in which he demonstrated
that two beams of light arriving simultaneously at a X
point may produce darkness. Young allowed O
Sunlight to fall on a pin hole S in a screen A. Light S1 Z
S2
emerging from S fell on two pin holes S1 and S2 in
C
another screen B. S1 and S2 acted as two sources of d D
light and light emerging from them overlapped in the S B
region XY of the screen C.
Features of interference pattern in Young’s A
double slit experiment Young’s experiment
(a) Alternate dark and bright fringes are present.
(b) Bright fringes and dark fringes are equally spaced.
(c) Bright fringes are equally bright.
(d) Dark fringes are equally dark.
46. Assuming the expression for the path difference in Young’s experiment, obtain an expression for fringe
width in the case of (a) dark fringes (b) bright fringes.
Solution
xd
Path difference, S2P − S1P =
D
For maximum, S2P − S1P = nλ
xd
Thus, nλ =
D
nλ D
Or, x = x n = , n = 0, ± 1, ± 2, ± 3, … [For maxima]
d
λ
Now, for minimum, S2P − S1P = (2n − 1)
2
λ xd
Thus (2n − 1) =
2 D
λD
or x = x n = (2n − 1) , n = ±1, ±2, ±3, … [For minima]
2d
The dark and bright fringes are equally spaced.
The fringewidth is the distance between two consecutive bright fringes or dark
fringes. (Fringewidth is not width of a fringe!)
Let nth order bright fringe is at a distance xn and (n + 1)th order bright fringe is
at xn +1 from O.
nλ D (n + 1)λD
Then x n = and x n +1 = (b) Fringe pattern
d d
λD
Now the fringewidth is β = x n +1 − x n =
d
λD
Thus, the expression for fringewidth is β =
d

49
2PBBBCPM7AS

47. Two narrow parallel slits 0.5 mm apart are illuminated with light consisting of two wavelengths 600 nm
and 400 nm, to obtain interference pattern on a screen 1 m from the slits. Calculate the least distance of
the point from the central bright fringe where the bright fringes due to both the wavelengths coincide.
[2.4 mm]
Solution
λ1 = 600 nm ; λ 2 = 400 nm ; d = 0.5 mm, D = 1 m
If x is the distance from the central fringe where the bright fringes due to the wavelengths λ1 and λ 2
nλD n λ D n λ 600 3
coincide, then x = n1β1 = n 2β2 i.e., 1 1 = 2 2  2 = 1 = =
d d n1 λ 2 400 2
For x to be minimum, n 2 = 3 and n1 = 2
n1λ1D 2 × 600 × 10−9 × 1
∴ Distance, x = = = 2 × 1.2 × 10−3 = 2.4 mm
d 0.5 × 10−3
48. Light from a narrow slit passes through two parallel slits 0.2 cm apart. Find the wavelength of light if the
interference fringes on a screen one metre distant from the two slits are 0.3 mm apart. If the whole
4
apparatus is immersed in water of refractive index , find the change in the fringewidth.
3
[600 nm, 0.075 mm]
Solution
d = 0.2 × 10−2 m, D = 1 m, β = 0.3 × 10−3 , λ =?
λD β × d 0.3 × 10−3 × 0.2 × 10−2
β= λ= =
d D 1
λ = 0.06 × 10−5 = 600 × 10−9 m = 600 nm
β 0.3 × 10−3
β′ = = = 0.225 × 10−3 m
n 4
3
Change in fringewidth = β − β′ = ( 0.3 − 0.225 ) × 10−3 = 0.075× 10−3 m = 0.075 mm

49. A double slit experiment produces interference fringes for sodium light of wavelength 589 nm that are
0.4° apart. What is the angular fringe separation when the entire experimental arrangement is immersed
under water [RI. of water = 4/3]? [0.3°]
Solution
λair = λ1 = 589 nm, θ1 = 0.4°, nwater = 4/3, θ2 = ?
When the apparatus is in water,
λ  n n 
λ water = air ∵, n 2 = 2 = 1 
n water  n1 λ 2 
589
= nm = 441.8.nm
4 ( )
3
d sin θ = mλ, m = 1, 2 …… Since sin θ ≈ θ, dθ ≃ mλ
θ λ
Thus, for a given order of the fringe, 1 = 1
θ2 λ 2
π
0.4° ×
 180° = 589  θ = 441.8 × 0.4° = 0.3°
2
π 441.8 589
θ2 ×
180°

50
2PBBBCPM7AS

Long answer type questions (LA)


50. Using Huygens’ principle, for refraction at a plane surface obtain Snell’s law.
Solution
Refer Module 7A
51. Using Huygens’ principle, for reflection at a plane surface prove laws of reflection.
Solution
Refer Module 7A
52. Obtain the expression for intensity at a point due to coherent addition of waves. Mention the conditions
for constructive interference and destructive interference.
Solution
Refer Module 7A
53. Obtain an expression for the fringewidth in Young’s double slit experiment.
[March 2014][June 2014][March 2018] [March 2020]
Solution
Refer Module 7A

Classwork Problems
54. A spectral line received from a distant star for a given element is shifted towards the red end by 3%. Find
the velocity of the star along the line of sight. [9 × 106 ms−1]
Solution
∆λ
Given: = 0.03
λ
∆ν
The velocity of the star is given by v =
ν
∆ν ∆λ
We have = ( −)
ν λ
∆λ
v=c = 3 × 108 × 0.03 = 9 × 106 m s −1
λ
The star is moving away from the observer.
55. The path difference between two beams of light reaching a point in a Young’s experiment is
1.2 × 10−6 m. Find the nature of interference at the point for light of wavelength (i) 600 nm
(ii) 480 nm and (iii) 540 nm. [(i) constructive (ii) destructive (iii) neither]
Solution
λ1 = 600 nm, λ2 = 480 nm, λ3 = 540 nm. Path difference, ∆x = 1.2 × 10−6 m
Case (i): When λ = 600 nm = 600 × 10−9 m,
∆ x 1.2 × 10−6
= = 5 i.e., ∆ = 5λ
λ 600 × 10−9
Thus, n = 5 which is an integer. Thus, the interference is constructive. (Bright point)
Case (ii): When λ = 480 × 10−9 m
∆ x 1.2 × 10−6
= = 2.5
λ 480 × 10−9
λ
∆ x = 2.5λ = 5  
2

51
2PBBBCPM7AS

Since path difference is an odd multiple of (λ/2) the point corresponds to destructive
interference (Dark point).
Case (iii): When λ = 540 nm = 540 × 10−9 m,
∆ x 1.2 × 10−6
= = 2.22  ∆ x = 2.22 λ
λ 540 × 10−9
Since ∆x is neither an integral multiple of λ nor an odd multiple of (λ/2), the point is neither an
interference maximum nor a minimum.
56. Calculate the angular positions of (a) 3rd bright fringe and (b) 5th dark fringe in an interference pattern
produced by light of wavelength 480 nm, if the slit separation is 0.16 mm. [0.009 rad, 0.0135 rad]
Solution
(a) d sin θ = nλ
For 3rd bright fringe, n = 3
nλ 3 × 480 × 10−9
sin θ = = −3
= 3 × 30 × 10−4 = 90 × 10−4 = 0.009
d 0.16 × 10
θ ≃ 0.009 rad
(2n − 1)
(b) For 5th dark fringe n = 5 , d sin θ = λ
2
(2 × 5 − 1) × 480 × 10−9 9 × 480
sin θ = = × 10−6 = 0.0135
2 × 0.16 × 10−3 2 × 0.16
∴θ ≃ 0.0135 rad
57. The distance between two coherent sources is 3 mm and the screen is 0.9 m from the sources. The
16th dark fringe is at a distance 2 mm from the central bright fringe. Find the
(a) wavelength of the light used
(b) distance of the 10th bright fringe from the central bright fringe.
(c) distance of the tenth bright fringe from the central bright fringe when the separation between the
screen and the slits is increased by 0.1 m. [(a) 430 mm, (b) 1.2 mm, (c) 1.433 mm]
Solution
(a) d = 3 mm = 3 × 10–3 m, D = 0.9 m, (x16)dark = 2 mm = 2 × 10−3 m, λ = ?
D λ
For a dark fringe, x n = ( 2n − 1) , n = 1, 2, ……
d 2
th
For the 16 dark fringe, n = 16
∴ 2 × 10−4 =
0.9
−3
( 2 × 16 − 1) × λ
3 × 10 2
−3 −3
2 × 10 × 3 × 10 × 2
λ= = 430 × 10−9 m = 430 nm
0.9 × 31
(b) To find the distance of the 10th bright fringe, λ = 430 nm and n = 10.
We know for a bright fringe at a distance x from the central bright fringe,
nλ D 10 × 430 × 10−9 × 0.9
For a bright fringe, x n = ∴ x10 = = 1.2 × 10−3 m
d 3 × 10−3
(c) To find the distance of 10th bright fringe when the separation between the screen and the slits is
increased by 0.1 m.
D = 0.9 + 0.1 = 1 m, n = 10
nλ D 10 × 430 × 10−9 × 1
x=  xn = −3
= 1.433 × 10−3 m
d 3 × 10

52
2PBBBCPM7AS

58. A beam of light consisting of two wavelengths 4200 Å and 5600 Å is used to obtain interference fringes
in YDSE. The distance between the slits is 0.3 mm and the distance between the slits and the screen is
1.5 m. Compute the least distance of the point from the central maximum, where the bright fringes due to
both the wavelengths coincide. [2.4 × 10−3] [July 2015]
Solution
• λ1 = 4200 Å, λ2 = 5600 Å, D = 1.5 m, d = 0.3 × 10−3 m,
When mth bright fringe due to wavelength λ1, coincides with nth bright fringe due to wavelength λ2,
Dmλ1 Dnλ 2
=
d d
m λ2
=
n λ1
m 5600(Å) 4
= =
n 4200 (Å) 3
To compute the least distance (from the central maximum) where the bright fringes coincides
m = 4 and n = 3
Dmλ1
• χλm1 =
d
1.5 × 4 × 4200 × 10−10
= −3
= 8.4 × 10−3 m OR
0.3 × 10
Dmλ 2 1.5 × 3 × 5600 × 10−10
χλn 2 = = = 8.4 × 10−3 m
d 0.3 × 10−3
59. In a YDSE, the distance between the slits is 1 mm. The fringewidth is found to be 0.6 mm. When the
screen is moved through a distance of 0.25 m, away from the plane of the slits the fringewidth becomes
0.75 mm. Find the wavelength of the light used. [6000 Å][March 2015]
Solution
d = 10−3 m, β1 = 0.6 × 10−3 m, D1 = D metre, λ = ?, β2 = 0.75 × 10−3 m, D2 = (D + 0.25) m

• β=
d
β2 (D + 0.25)
=
β1 D
0.75 × 10−3 (D + 0.25)
=
0.6 × 10−3 D
D + 0.25 5
=
D 4
4D + 1 = 5D  D = 1 m
1× λ
• 0.6 × 10−3 = −3
10
λ = 0.6 × 10−6 m = 600 nm = 6000 Å
OR
1.25 × λ
0.75 × 10−3 = (D + 0.25 = 1 + 0.25 = 1.25 m)
10−3
0.75 ×10−6
λ= = 0.6 ×10−6 m = 6000 Å
1.25

53
2PBBBCPM7AS

60. In Young’s double slit experiment, fringes of certain width are produced on the screen kept at a certain
distance from the slits. When the screen is moved away from the slits by 0.1 m, fringewidth increases by
6 × 10−5 m. The separation between the slits is 1 mm, calculate the wavelength of the light used.
[6000 Å][March, 2016]
(Similar: d = 0.28 mm, D = 1.4 m, x 5dark = 1.35 cm , λ = ?); If D′ = 1 m, β = ? [600 nm; 2.14 × 10−3 m]
[March 2018]
Solution
β1 = β metre, D1 = D metre, d = 10−3 m
Similar
β2 = (β + 6 × 10−5)m, D2 = (D + 0.1)m, λ = ?
Hint:

β= …(1) D λ
d x dark
r = (2n − 1) , put n = 5
d 2
(D + 0.1)λ
(β + 6 × 10−5 ) = …(2) λ = 600 nm
d
D′
0.1λ β = λ = 2.1 × 10−3 m
Equation (2) − equation (1)  6 × 10−5 = d
d
6 × 10−5 × 10−3
λ= = 600 nm = 6000 Å
0.1
61. In YDSE, while using a source of light of wavelength 6000 Å, the fringewidth was found to be 6 mm. If
the distance between the screen and the plane of the slits is reduced to half, what should be the
wavelength of light to get fringewidth of 4 mm? [8000 Å] [March 2017]
Solution
D
β1 = 5 × 10−3 m, D1 = D, λ1 = 6000 Å, β2 = 4 × 10−3 m, D 2 = , λ2 = ?
2

β=
d
β1 Dλ1 6 × 10−3 2 × 6000 (Å)
=  =
β2  D  4 × 10−3 λ2
  2 λ
 
2
4 × 2 × 6000 (Å)
λ2 = = 8000 Å
6
62. In a YDSE, wavelength of light used is 5000 Å and the distance between the slits is 2 mm, distance of
the screen from the slits is 1 m. Find fringewidth. Also calculate the distance of 7th dark fringe from the
central bright fringe [0.25 × 10−3 m, 1.625 × 10−3 m] [March 2019]
Solution
–10
λ = 5000 Å = 5000 × 10 m,
D = 2 mm = 2 × 10–3 m, n = 7
D = 1 m, β = ?, x dark
7 =?
Dλ 1 × 5000 × 10−10
β= = = 2.5 × 10−4 m
d 2 × 10−3
Dλ  2n − 1   2n − 1 
x dark
n =  ≃ 
d  2   2 
 2 × 7 −1 13
x dark
7 = 2.5 × 10−4  −4
 = 2.5 × 10 × = 1.625 × 10–3 m
 2  2

54
2PBBBCPM7AS

Practice Exercise
63. In Young’s double slit experiment, the distance between two coherent sources is 1 mm and the distance
between the slits and the screen is 1 m. If the 10th dark fringe is at a distance of 4.75 mm from the central
bright fringe, calculate
(a) the wavelength of light used and
(b) the distance of 5th bright fringe from the central bright fringe. [500 nm, 2.5 mm]
Solution
d = 1 mm; D = 1 m ; for 10th dark fringe n = 10 ; x n = 4.75 mm . For 5th bright fringe, n = 5
(2n − 1) (2n − 1) λD
(a) For nth dark fringe, x n = β  xn =
2 2 d
(2 × 10 − 1) λ × 1
4.75 × 10−3 =
2 1 × 10−3
19λ
4.75 × 10−3 =
2 × 10−3
9.5
λ= × 10−6 = 0.5 × 10−6 = 500 nm
19
(b) For fifth bright fringe, n = 5 . Distance from central fringe, x n = nβ
nλD 5 × 500 × 10−9 × 1
xn = = = 2500 × 10−6 = 2.5 mm
d 1 × 10−3
64. In a Young’s double slit experiment, two narrow parallel slits 0.1 mm apart are illuminated with
monochromatic light of wavelength 5890 Å to get fringe pattern on a screen 1 m away from the slits.
Calculate the distance of
(i) fifth dark fringe from the central bright fringe and
(ii) third bright fringe from the central fringe. [(i) 26.5 mm, (ii) 17.67 mm]
Solution
Distance between the n th dark fringe and central bright fringe,
(2n − 1)λD
xn = (Consider n = 5, for 5th dark fringe)
2d
(2 × 5 − 1) × 5890 × 10−10 × 1
x5 = = 26.5 mm
2 × 0.1 × 10−3
nλ D
Distance between n th bright fringe and central bright fringe, x n =
d
−10
3 × 5890 × 10 × 1
x3 = = 17.67 mm
0.1 × 10−3
65. Two narrow parallel slits 0.5 mm apart are illuminated with light consisting of two wavelengths 700 nm
and 350 nm, to obtain interference pattern on a screen 2 m from the slits. Calculate the least distance of
the point from the central bright fringe where the bright fringes due to both the wavelengths coincide.
[2.8 mm]
Solution
λ1 = 700 nm, λ 2 = 350 nm, d = 0.5 mm, D = 2 m
xn = xp
n1λ1D n 2 λ 2 D λ n
=  1 = 2
d d λ 2 n1

55
2PBBBCPM7AS

700 n 2 n 2
=  2 =  n 2 = 2 and n1 = 1
350 n1 n1 1
n1λ1D 1 × 700 × 10−9 × 2
∴ xn = =
d 0.5 × 10−3
x n = 2.8 mm
66. In a Young’s double slit arrangement, the slits are 1.5 mm apart. When the slits are illuminated by a
monochromatic source and the screen is kept 1 m apart from the slit, the width of 10 fringes are
measured to be 3.93 mm. Calculate the wavelength of light used. What will be the width of 10 fringes
when the distance between the slits and the screen is increased by 0.5 m when the same source of length
is used? [5895 Å, 5.9 µm]
Solution
10β = 3.93 × 10−3 m, D = 1 m , λ = ?
β = 3.93 × 10−4 m
λD β d 3.93 × 10−4 × 1.5 × 10−3
β= λ= =
d D 1
λ = 5.895 × 10−7 = 5895 Å
λ × 1.5 5895 × 10−10 × 1.5
β′ = =
1.5 × 10−3 1.5 × 10−3
β′ = 5895 × 10−7 10 β′ = 5895 × 10−6 = 5.895 µ m
67. In a Young’s double slit experiment, the wavelength of light used is 500 nm, slit separation is
1 mm and the distance between the slits and the screen is 1.2 m. Calculate the distance between 5th and
10th bright fringes. [3 mm]
Solution
λ = 500 nm, d = 1 mm, D = 1.2 m, n1 = 5, n 2 = 10
Distance of n1th bright fringe from the central fringe, x1 = n1β
Distance of n 2 th bright fringe from the central fringe, x 2 = n 2β
∴ Distance between these two fringes,
d = x 2 − x1 = (n 2 − n1 )β
λD 5 × 500 × 10−9 × 1.2
= (10 − 5) = −3
= 2500 × 1.2 × 10−6 = 3000 × 10−6 = 3 mm
d 1 × 10
Distance between 5th and 10th bright fringe = 3 mm
68. The wavelength of Hγ line in radiation from a distant galaxy is measured to be 462 nm. The wavelength
of Hγ line is 434 nm when observed in the laboratory. (a) What is the speed of the galaxy in the line of
sight relative to earth? (b) Is the galaxy approaching or receding? [(a) 1.94 × 107 ms−1 (b) receding]
Solution
∆ν
The velocity of the galaxy is given by v = c
ν
∆ν ∆λ
= ∆λ = 462 nm − 434 nm=28 nm
ν λ
∆λ 3 × 108 × 28 × 10−9
v=c = −7
= 1.94 × 107 m s −1
λ 4.34 × 10
Increase in the wavelength (red shift) indicates that the galaxy is moving away from us.

56
2PBBBCPM7AS

69. In double-slit experiment using light of wavelength 600 nm, the angular width of a fringe formed on a
distant screen is 0.1°. What is the spacing between the two slits? [0.344 mm]
Solution
Given, wavelength of light λ = 600 nm = 600 × 10−9 m
0.1π
Angular width of fringe θ = 0.1° = rad
180
λ λ 600 × 10−9 × 180
Using the formula θ = , spacing between the slits is d = =
d θ 0.1 × π
−4
d = 3.44 × 10 m = 0.344 mm
70. In Young’s double slit experiment distance between the slits is 0.5 mm. When the screen is kept at a
distance of 100 cm from the slits, the distance of ninth bright fringe from the center of the fringe system
is 8.835 mm. Find the wavelength of light used. [490.8 nm], [June 2017]
Solution
d = 0.5 mm, D = 100 cm, x gb = 8.835 mm , λ = ?
nDλ 9dλ
x bn = x gb =
d d
x gb d 8.835 × 10−3 × 0.5 × 10−3
λ= =
9D 9 ×1
λ = 490.8 nm
71. In YDSE, while using a source of light of wavelength 4500 Å, the fringewidth was found to be 5 mm. If
the distance between the screen and the plane of the slits is reduced to half, what should be the
wavelength of light to get fringewidth of 4 mm? [7200 Å] [June 2016]
Solution
D
β1 = 5 × 10−3 m, D1 = D, λ1 = 4500 Å, β 2 = 4 × 10−3 m, D 2 = , λ2 = ?
2

β=
d
β1 Dλ1
=
β2  D 
  λ2
2
5 × 10−3 2 × 4500 (Å)
=
4 × 10−3 λ2
4 × 2 × 4500 (Å)
λ2 = = 720 nm = 7200 Å
5

10.2. Diffraction
Exercise
Very short answer questions (VSA)
1. What is meant by diffraction of light?
Solution
Diffraction of light is the phenomenon of bending of light around the edges of an obstacle whose size is
comparable to the wavelength of light.

57
2PBBBCPM7AS

2. When are diffraction effects observable?


Solution
Diffraction effects are observable when the dimension of an obstacle is comparable to the wavelength of
light.
3. Why is diffraction of sound easier to observe than diffraction of light?
Solution
The wavelength of sound waves is very large compared to that of light waves and hence comparable
with the dimensions of obstacles.
4. Why diffraction of light is not easily noticed in our daily life?
Solution
To observe diffraction of light, the dimensions of the obstacles should be comparable with the
wavelength of light. Since, the wavelength of light is very much less than the dimensions of obstacles,
under ordinary conditions, diffraction of light is not observed.
5. Draw the intensity distribution curve in the case of single slit diffraction.
Solution
Intensity

Principal maximum

First secondary
maximum First minimum

Distance from
the centre
6. Why the centre of the diffraction pattern due to a single slit is has maximum intensity?
Solution
Wavelets from all the points of the diffracted wavefront reinforce in phase to give maximum intensity at
the centre of the diffraction pattern.
7. Mention the expression for the width of the central maximum in the diffraction pattern due to a single
slit.
Solution
The expression is,
2λD
β= , where, λ is the wavelength of light, D is the distance between the slit and the screen and a is
a
the width of the slit.
8. State the condition for secondary maximum in the single slit diffraction experiment.
Solution
λ
The condition for secondary maxima is that path difference, a sin θ = (2n + 1) , where n = integer,
2
a = width of the sit, λ = wavelength of light, θ = angle of diffraction.
9. State the condition for the minima in a single slit diffraction experiment.
Solution
For secondary minima, path difference a sin θ = nλ, where the symbols have the usual meaning.

58
2PBBBCPM7AS

10. What is Fresnel’s distance?


Solution
Fresnel’s distance is defined as the distance of the screen from a slit at which the spreading of light due
to diffraction becomes equal to the size of the slit. It is denoted by Zf.
11. What is resolving power of an optical instrument?
Solution
Resolving power of an optical instrument is its ability to show two close lying points on an object as well
separated points in the image.
12. Define limit of resolution of a microscope.
Solution
The limit of resolution of a microscope is defined as the minimum separation between two point objects
so that they are seen as just separated in the image.
13. Define limit of resolution of a telescope.
Solution
Limit of resolution of a telescope is defined as the smallest angle that has to be subtended by two point
objects at the objective so that they are seen as just separated in the image.
14. Define resolving power of a microscope.
Solution
The resolving power of a microscope is defined as the reciprocal of the minimum separation between
two point objects so that they are seen as just separated in the image.
15. What is the value of limit of resolution of human eye?
Solution
The limit of resolution of human eye is about one arc minute.
16. How does the resolving power of a microscope vary with the wavelength of light used?
Solution
Resolving power of a microscope is inversely proportional to the wavelength of the light.
17. How does the resolving power of a microscope vary with the refractive index of the medium between the
objective and the object?
Solution
Resolving power of a microscope increases with increase in the refractive index of the medium between
the object and objective.
18. Mention a method of increasing the resolving power of a telescope. [March 2016]
Solution
The resolving power of a telescope can be increased by increasing the aperture of the lens.
19. Write the relation between resolving power of a telescope and limit of resolution.
Solution
Resolving power of a telescope is the reciprocal of its limit of resolution.
20. How does the resolving power of a telescope change if the wavelength of light is decreased?
Solution
Increases.

59
2PBBBCPM7AS

21. Two microscopes have limits of resolution of 10 µm and 15 µm respectively. Which one of them has a
higher resolving power?
Solution
Microscope with 10 µm limit of resolution has higher resolving power.
22. The objectives of two telescopes have diameters of 10 cm and 15 cm respectively. Which of them has a
higher resolving power?
Solution
Telescope with 15 cm objective.

Short answer questions (SA - 2 M)


23. Mention any two features of diffraction bands in a single slit diffraction pattern.
Solution
(i) It has alternate dark and bright bands of continuously decreasing width symmetrically situated on
both sides of the central bright band.
(ii) As we move away from the central bright band, bright bands gradually become less bright and the
dark bands become less dark until they become indistinguishable.
24. Write any two differences between interference fringes and diffraction bands.
Solution
Interference pattern (YDSE) Diffraction pattern (single slit)
1. The interference pattern has a number of 1. The diffraction pattern has a central bright
equally spaced bright and dark bands. maximum which is twice as wide as the first
secondary maximum.
2. The intensity of bright bands is almost 2. The intensity of bright bands decreases as we
same. go to successive maxima, on either side.
25. Estimate the distance for which ray optics is good approximation for an aperture of 4 mm and
wavelength 400 nm.
Solution
a 2 (4 × 10−3 ) 2
Fresnel distance, Zf = = = 40 m
λ 400 × 10−9
26. Explain methods to improve the resolving power of a microscope.
Solution
Resolving power can be increased by (i) decreasing λ (Example: when ultraviolet light is used to
illuminate an object, it is called ultramicroscope) (ii) increasing n (Example: In an oil immersion
objective type, oil of suitable refractive index is used to increase the resolving power). In an electron
microscope, electron beam consisting of electrons having de Broglie wavelength 104 times smaller than
wavelength of visible light is used, thereby increasing the resolving power 104 times.

60
2PBBBCPM7AS

Short answer type questions (SA - 3 M)


27. A convex lens of focal length 0.4m forms a single slit diffraction pattern of a slit 0.02 mm wide.
Calculate the distance of the third bright band from the central maximum, if the wavelength of light used
is 690 nm. [48.3 mm]
Solution
Wave length, λ = 690 nm = 690 × 10–9 m, focal length of lens, f = 0.4 m, slit width,
a = 0.02 mm = 2 × 10–5 m
λ
Condition for secondary maxima, a sin θ = ( 2n + 1) where n = 1, 2, 3, ……,
2
7λ 7 × 6.9 × 10−7
For third secondary maximum, n = 3 and hence, sin θ = = = 0.1208
2a 2 × 2 × 10−5
θ = 0.1208 rad (since θ is very small).
Distance from the central maximum, x = fθ = 0.4 × 0.1208 = 0.04832 m = 4.83 × 10−2 m = 48.3 mm
28. In a single slit diffraction experiment when light of wavelength 589.3 nm is used, the distance between
the two first order minima on both the sides of the central maximum is 0.028 m. The distance of the
screen is 1 m from the plane of the slit. Calculate the width of the slit. [4.2 × 10–5 m]
Solution
Wave length, λ = 589.3 nm = 5.893 × 10–7 m, D = 1 m
Linear width of central maximum = 0.028 mm = 28 × 10–6 m, a = ?
2λ D
Width of central maximum =
a
2 × 5.893 × 10−7 × 1
∴ a= = 4.2 × 10−5 m
28 × 10−6
29. Light of wavelength 560 nm falls normally on a slit of width 0.05 mm. Find the angular positions of
third secondary maximum and third minimum in the diffraction pattern. [0.0392 rad, 0.0336 rad]
Solution
Wave length, λ = 560 nm = 560 × 10–9 m, slit width a = 0.05 mm = 5 × 10–5 m
(2n + 1)λ
Condition for secondary maxima, a sin θ = where n = 1,2,3, . . .
2
For third secondary maximum, n = 3
7λ 7 × 5.60 × 10−7
sin θ = = = 0.0392  θ ≃ 0.0392 rad
2a 2 × 5 × 10−5
Condition for minima, a sin θ = nλ
3λ 3 × 5.6 × 10−7
For third minimum, n = 3, sin θ = = = 0.0336  θ ≃ 0.0336 rad
a 5 × 10−5

Long answer questions (5 M)


30. For diffraction at a single slit, obtain to the conditions for minima and maxima.
Solution
Refer Module 7A
31. What is resolving power of a microscope? Explain using the concept of Airy disk.
Solution
Refer Module 7A

61
2PBBBCPM7AS

32. Obtain an expression for limit of resolution and resolving power of a compound microscope.
Solution
Refer Module 7A
33. Define limit of resolution and resolving power of a telescope. Write the expression for limit of the
resolution and resolving power of a telescope. How the resolving power of a telescope can be improved?
Solution
Refer Module 7A
34. Write the expression for limit of resolution of (a) microscope (b) telescope. Mention a method of
increasing the resolving power of a microscope.
Solution
Refer Module 7A

Class work Problems


35. A parallel beam of light of wavelength 600 nm is incident on a diffracting aperture of width 3 mm wide.
Estimate the distance upto which ray optics is a good approximation. [15 m]
Solution
a2
Fresnel distance is given by Zf ≃ .
λ
(3 × 10−3 )2
Hence, Zf = = 15 m
6 × 10−7
36. A beam of parallel rays of light of wavelength 600 nm is incident normally on a slit. Due to diffraction,
each first order minimum lies at a distance of 0.6 mm from the central maximum on either side of it. The
distance of the screen from the slit is 1 m. Calculate the width of the slit. [1 mm]
Solution
In single slit diffraction, condition for minima is a sin θ = nλ. When θ is small and is in radian,

θ (rad) ≈ . If x is the distance of the first minimum from the central maximum and D is the
a
x
distance of the screen from the slit, θ (rad) ≈ .
D
Substituting the values of n, λ, x and D, width of the slit (a) can be calculated.
For the first minimum, n = 1 First minimum
∴ a sin θ = λ x
λ q
When θ is small and is in radian, sin θ ≈ θ = Central maximum
a D
600 × 10−9
∴θ = …(1)
a
If x is the distance of the first minimum from the central maximum and D is the distance of the screen
from the slit,
x
tan θ ≈ θ =
D
−3
0.6 × 10
θ= …(2)
1
600 × 10−9 0.6 × 10−3
From (1) and (2), = ∴ a = 1000 × 10−6 = 1 × 10−3 m = 1 mm
a 1

62
2PBBBCPM7AS

37. Find the separation between two points on the moon that can be resolved by a 5 m telescope. The
distance of moon from the earth is 3.8 × 105 km. The eye is most sensitive to light of wavelength
5500 Å. [50.99 m]
Solution
Objective diameter, a = 5 m. Earth - Moon distance, d = 3.8 × 105 m
1.22λ 1.22 × 5500 × 10−10
∆θ = = = 1342 × 10−10 radian
a 5
Let y be the separation between the points S1 and S2 on the moon.
y = ∆θ × d
y = 1342 × 10−10 × 3.8 × 105 × 103  y = 5099.6 × 10−2 y = 50.99 m
38. While using a microscope it is found that the semi vertical angle is 15°. Calculate the limit of resolution
and resolving power when light of wavelength 540 nm illuminates the specimen. [7.87 × 105]
Solution
Given θ = 15°, λ = 540 nm, for air n = 1
Limit of resolution of the microscope for non-luminous objects is
1.22λ 1.22 × 540 × 10−9 1.22 × 540 × 10−9
dx = = = = 1.27 × 10−6 m
2 n sin θ 2 × 1 × sin15° 2 × 1 × 0.2588
1 1
Resolving power = = −6
= 7.87 × 105
dx 1.27 × 10

Practice Exercise
39. Estimate the distance for which ray optics is good approximation for an aperture of 4 mm and
wavelength 400 nm. [40 m]
Solution
a 2 (4 × 10−3 ) 2
Fresnel distance, Zf = = = 40 m
λ 400 × 10−9
40. Two towers on top of two hills are 40 km apart. The line joining them passes 50 m above a hill halfway
between the towers. What is the longest wavelength of radio waves which can be sent between the
towers without appreciable diffraction effects? [0.125 m]
Solution
a2
Size of aperture, a = 50 m Fresnel’s distance, ZF =
λ
2
50
ZF = 20 km, Hence, λ = = 0.125 m
20 × 103
41. A parallel beam of light of wavelength 500 nm falls on a narrow slit and the resulting diffraction pattern
is observed on a screen 1 m away. It is observed that the first minimum is at a distance of 2.5 mm from
the centre of the screen. Find the width of the slit. [0.2 mm]
Solution
λ = 500 nm, D = 1 m, x1 = 2.5 mm
a sin θ = nλ for minima
∴ a θ = λ for first minimum as θ = small
λ λ λD 5 × 10−7 × 1
θ = , x1 = θD = D a = = = 2 × 10−4 m = 0.2 mm
a a x1 2.5 × 10−3

63
2PBBBCPM7AS

42. The semi vertical angle subtended by two points on an object at the objective of a microscope is 10°. The
object is illuminated using light of wavelength 600 nm. Calculate the minimum distance between two the
points at which they are just resolved. Comment on the improvement, if the microscope is of oil
immersion type. The refractive index of oil used is 1.5.
[2.11 µm, 1.41 µm: There is an improvement in resolving]
Solution
Wavelength, λ = 600 nm = 6 × 10–7 m
When the space between the object and the objective contains air (vacuum), limit of resolution of the
1.22λ
microscope is given by ( dx )1 =
2sin θ
−7
1.22λ 1.22 × 6 × 10
dx = =
2n sin θ 2 × 1 × sin10°
1.22 × 6 × 10−7
= = 2.11 × 10−6 m =2.11 µm
2 × 1 × 0.1736
When the space between the object and the objective is filled with oil (n = 1.5), the limit of resolution of
1.22λ
the microscope is given by ( dx )2 =
2n sin θ
−6
dx 2.11 × 10
When n = 1.5, dx ′ = = = 1.41 × 10−6 m = 1.41 µm
n 1.5
Limit of resolution decreases from 2.11 µm to 1.41 µm.
Hence, resolving power increases 1.5 times the initial value.
43. Calculate the aperture of the objective of a telescope which can be used to just resolve two stars
separated by 6.1 × 10−6 radian, if light from the star has a wavelength of 550 nm. [0.11 m]
Solution
Limit of resolution, ∆θ = 6.1 × 10–6 radian
Wavelength, λ = 550 nm = 5.5 × 10–7 m
1.22λ 1.22 × 5.5 × 10−7
∆θ = a = = 0.11 m
a 6.1 × 10−6

10.3 Polarisation
Exercise
Very short answer questions (VSA)
1. What is polarisation of light?
Solution
Polarisation is the process of eliminating the symmetric distribution of electric field vectors around the
line of propagation of light, so that electric field vectors in one plane is retained.
2. What is plane of polarisation?
Solution
The plane containing the electric field vectors.
3. What is linearly polarised light?
Solution
In linearly polarised light (often called plane polarised light) the electric field vectors E oscillate in a
fixed plane. (The projection of the tip of electric field vector on a plane perpendicular to the line of
propagation is a straight line).

64
2PBBBCPM7AS

4. What is polarising angle?


Solution
It is the angle of incidence for which the reflected and refracted rays are perpendicular to each other, and
the reflected light is completely plane polarised.
5. State Malus’ law
Solution
When unpolarised light of intensity Io is passed through a polaroid P1 (polariser), the transmitted light
Io
which is plane polarised will have an intensity, I′ = . If this polarised light is passed through a second
2
polaroid P2 (analyser), the intensity of light transmitted by second polaroid depends on the orientation θ,
the angle between the pass – axes, of P1 and P2. The intensity of light transmitted by P2 is given by
I = I′ cos2 θ. This is called Malus’ law.
6. What is the angle between the reflected and refracted rays for light incident on a reflecting medium at
polarising angle?
Solution
90° or π / 2 radians.
7. State Brewster’s law.
Solution
Brewster’s law states that for a given wavelength of light, the refractive index of a medium is equal to
the tangent of the polarising angle.
8. What is the relation between the angle of refraction and polarising angle?
Solution
r = 90 ° − θP .
9. What is the Brewster’s angle for air to glass transmission? (Refractive index of glass = 1.5)
Solution
From Brewster’s law, we have n = tan θP  θP = tan−1 (1.5) = 56° 18′

Short answer questions (SA - 2 M)


10. Explain the working of a polaroid.
Solution
Polaroids are thin sheets of specially prepared plastic material which are generally used to get plane
polarised light.
Hence, when unpolarised light is passed through a Polaroid, we get linearly polarised light with
E - vector oscillating along a direction perpendicular to the aligned molecules. This direction is known
as the pass-axis or transmission axis of the polaroid.
11. What is pass-axis of a polaroid?
Solution
If an unpolarised light wave is incident on a polaroid, then, the light wave will get linearly polarised with
the electric vector oscillating along a direction perpendicular to the aligned molecules; this direction is
known as the pass-axis of the polaroid.

65
2PBBBCPM7AS

12. Mention any two applications of polaroids.


Solution
Polaroids are used in sun glasses, window panes of aeroplanes, cameras, microscope objectives to
eliminate glare of reflected light and to view 3-D pictures and movies.
13. State and explain Brewster’s law.
Solution
The tangent of the polarising angle is equal to the relative refractive index of the medium 2 w.r.t.
medium 1, for a given wavelength tan θ p = n 21 (tan i B = n 21 ) This is known as Brewster’s law.

14. A ray of light incident on a glass plate at the polarising angle gets deviated by 24° on entering the glass.
Calculate the polarising angle for glass. [57°]
Solution
i p − r = 24o
i p + r = 90o
2i p = 114o  i p = 57 o

15. What is the value of Brewster’s angle when light goes from air to glass ?
(Refractive index of glass = 1.5) [56° 18′]
Solution
From Brewster’s law, we have
n = tan θP  θP = tan−1 (1.5) = 56° 18′

Short answer type questions (SA - 3 marks)


16. Explain Malu’s law with the help of a diagram.
Solution
When unpolarised light of intensity Io is passed through a polaroid P1 (polariser), the transmitted light
Io
which is plane polarised will have an intensity, I′ = . If this polarised light is passed through a second
2
polaroid P2 (analyser), the intensity of light transmitted by second polaroid depends on the orientation θ,
the angle between the pass – axes, of P1 and P2. The intensity of light transmitted by P2 is given by I = I′
cos2 θ. This is called Malus’ law.

I0 I′ = I0/2

I = I′ cos2θ

Intensity of emergent light from two polarisers with transmission axes inclined at angle θ - Malus’ law.

66
2PBBBCPM7AS

17. Light of wavelength 5000 Å falls on a plane reflecting surface. What are the wavelength and frequency
of the reflected light? For what angle of incidence is the reflected ray normal to the incident ray?
Solution
Wavelength of reflected light = 5000 Å.
c 3 × 108
Frequency of reflected light is, ν = = = 6 × 1014 Hz
λ 0 5 × 10−7
Angle of incidence = angle of reflection. If the reflected ray has to be normal to the incident ray, i = 45°.
18. The critical angle of incidence for water is 48° for a certain wavelength. What is the polarising angle and
the angle of refraction for light incident on water at an angle that gives maximum polarisation of
reflected light? [53° 23′, 36° 37′]
Solution
1 1
nw = = = 1.346
sin C sin 48o
n w = tan i p  i p = tan −1 (1.346) = 53o 23′
i p + r = 90o  r = 90o − 53o 23′ = 36o 37′

19. A beam of light is incident from air on a glass slab at an angle 57°. The reflected and refracted beams
are perpendicular to each other. Calculate the speed of light in glass. [1.95 × 108 m s−1]
Solution
n = tan θP
c 3 × 108
n = = tan 57 o = 1.54 v= = 1.95 × 108 m s -1
v 1.54
20. Explain polarization of sunlight by scattering with the help of a diagram.
Solution
The light from a clear blue portion of the sky shows a rise and fall of
intensity when viewed through a polaroid which is rotated. This is
nothing but sunlight, which has changed its direction (having been
scattered) on encountering the molecules of the earth’s atmosphere. As
figure shows, the incident sunlight is unpolarised. The dots stand for
polarisation perpendicular to the plane of the figure. The double arrows
show polarisation in the plane of the figure. (There is no phase relation
between these two in unpolarised light). Under the influence of the
electric field of the incident wave the electrons in the molecules acquire
components of motion in both these directions. We have drawn an observer looking at 90° to the
direction of the sun. Clearly, charges accelerating parallel to the double arrows do not radiate energy
towards this observer since their acceleration has no transverse component. The radiation scattered by
the molecule is therefore represented by dots. It is polarised perpendicular to the plane of the figure. This
explains the polarisation of scattered light from the sky.

67
2PBBBCPM7AS

Long answer questions (5 M)


21. Define polarising angle. Show that n = tan θp, when light is incident at the polarising angle on a
refracting surface.
OR
Define polarising angle. State and prove Brewster’s law.
Solution
Refer Module 7A
22. State Brewster’s law. Show that the reflected and refracted rays are perpendicular to each other when the
angle of incidence is equal to the polarizing angle.
Solution
Refer Module 7A

Class work Problems


23. Two polaroids are placed in crossed position. One of them is rotated through 30°. What percentage of
incident unpolarised light will emerge out? [12.5%]
Solution
Initially, θ = 90°
I
• Intensity of light from the I-polaroid = 0
2
pola pola 2
• Iout = Iin cos θ
Now, the angle between the polaroids = θ = 60°
2
I0 I 1 I
Iout = cos 2 (60°) = 0   = 0 = 0.125 I0
2 2 2 8
24. Calculate the polarising angle for water-liquid interface. Given the refractive indices of liquid and water
are respectively 1.64 and 1.33. [50° 57′]
Solution
n liq 1.64
= tan θp  = tan θp = 1.233  θp  50°57′
nw 1.33
25. A ray of light travelling through water strikes a glass plate at an angle of 51°. A part of it is reflected and
the remaining is refracted. The reflected ray is found to be completely plane polarised. If the refractive
index of water is 1.33, calculate the refractive index of glass. [1.64]
Solution
Reflected ray is totally plane polarized. Hence, θp = 51°
θp = 51°, n w = 1.33
n
n gw = tan θp  g = tan θp
nw
n g = n w × tan θp
n g = 1.33 × tan(51°) = 1.64

68
2PBBBCPM7AS

Practice Exercise
26. Two polaroids are placed crossed to each other. Now one of them is rotated through 60°. What
percentage of incident unpolarised light will pass through the system? [37.5%]
Solution
I
After passing through first polaroid I1 = 0 ,
2
Initially the angle between the polaroids = 90°.
When one of the polaroids is rotated through 60°, angle between them = 30°.
I 3
After passing through second polaroid, I 2 = I1 cos 2 θ = 0 cos 2 30° = I0 = 37.5%
2 8
27. Given critical angle for a certain glass is 38° 42′ and that for glycerine is 42° 42′. Find the polarising
angle when light travels from glass to glycerine. [38° 42′]
Solution
1
n= ;Cgla < Cgly  n gla > n gly (glass is denser)
sin C
1
n gly sin C gly
n gly − gla = =
n gla 1
sin Cgla
sin Cgla sin 38°42′
= =
sin Cgly sin 42°42′
0.5402
= = 0.7966
0.5781
∴ ip = tan−1 (0.7966) = 38° 32′ (Glass → glycerine  denser to rarer  iP < 45° )
28. A ray of light incident on a glass slab at the polarising angle undergoes a deviation of 22°. Find the value
of angle of refraction in glass. [34°]
Solution
θp + r = 90° ...(1)
and d = i − r ...(2)
 r = i − d = θp − d ∴ θp + θp − d = 90°
90 + 22
2θp − 22° = 90° ; θp = = 56°
2
From (1), r = 90 − θp = 90 − 56 = 34° .

29. The polarising angle for water is 53° and that for glass is 57°. Calculate the polarising angle, when a
monochromatic ray of light goes from glass to water. [40° 48’]
Solution
n g = tan 57 = 1.54
n w = tan 53 = 1.33
(For glass: water interface), D R  θp < 45°
ng 1.33
n wg = = = 0.8636
nw 1.54
tan θ = 0.8636  θ = 40° 48′

69
2PBBBCPM7AS

30. The polarising angle for water-glass interface is 48° 26’. Find the critical angle. [62°° 26′′]
Solution
ng
= tan θp = tan 48°26′ = 1.128
nw
nw 1
sin C = = = 0.8865
n g 1.128
C = sin−1 (0.8865) = 62° 26′

Test Paper
PART - A
I. Answer the following questions [10 × 1 = 10]
1. A concave mirror (fair = 10 cm) is held in water. What will be the change in the focal length of the
mirror?
Solution
Focal length of the mirror remains same.
2. When light travels from a rarer to a denser medium, does the change in the speed imply a change in the
energy carried by the light wave? Justify your answer.
Solution
No. A change in the speed is accompanied by a change in the wavelength, such that the frequency
remains constant.
3. How does the angle of minimum deviation due to a glass prism vary, if the incident violet light is
replaced with red light?
Solution
Decreases.
4. Why rainbow is in the form of an arc?
Solution
Angle of incidence on nearby spherical raindrops from the position of observation is same along an arc
on a vertical plane. So an arc is observed.
5. How does the focal length of a lens for red light change when blue light is used?
Solution
As the refractive index of the material of a lens for red is less than that for blue, a parallel beam of light
incident on a lens will be bent more towards the axis for blue light compared to that for red. Thus the
focal length of the lens for blue light will be smaller than that for red.
6. What is optical tube length of a compound microscope?
Solution
The distance between the second focal point of the distance (Fo on the right hand side) and the first focal
point of the eyepiece (Fe on the left hand side) is called optical tube length of a compound microscope.
(It is nearly equal to vo).
7. What is Doppler effect in light?
Solution
The apparent change in frequency of waves from a source of light due to the relative motion between
and observer (detector) and a source is called Doppler effect.

70
2PBBBCPM7AS

8. What happens if in a Young’s double slit experiment if one of the slits is covered by an opaque paper?
Solution
Diffraction pattern is observed. (Fresnel type, because incident wave fronts are cylindrical)
9. How does the width of the central maximum in a single-slit diffraction experiment change when the
distance of separation between the slit and screen is doubled?
Solution
 2λ D 
On doubling the separation between the slit and the screen, the width of central maximum   is
 a 
doubled.
10. Unpolarised light of intensity I0 is passed through a polaroid. What is the intensity of the emerging light
from by the polaroid?
Solution
I
Iout = 0
2

PART - B
II. Answer any FIVE of the following questions [5 × 2 = 10]
11. A convex lens is immersed in water. What change, if any, do you expect in its focal length?
Solution
Focal length of a mirror is about half of its radius of curvature and has nothing to do with the external
medium. [1]
The focal length of the convex lens will increase because the refractive index of glass with respect to
water is less than the refractive index of glass with respect to air. [1]
12. Some motor cars have additional yellow headlights. Why?
Solution
Cars are provided with additional yellow headlights to have better visibility in foggy weather. [1]
We know that intensity of scattered light is inversely proportional to fourth power of wavelength (for a
given angle of observation). Hence, green light is scattered more than yellow light. Thus, yellow light
can penetrate through the fog and illuminate the road ahead, over a considerable distance. [1]
13. The percentage increase in the radius of curvature of each surface of an equi-convex lens is 20%.
Calculate the percentage change in its focal length.
Solution
1 2
= (n − 1)   …(1) [1]
f R
  20  
R ' = R +   R = R (1.2 ) 
  100  
1  2 
= ( a − 1)   …(2)
f'  R (1.2 ) 
(1) = f ' = R (1.2 ) 
f'
= 1 = 1.2 − 1 = 0.2
n f R f
∆f
∴ = 0.2  % change = 20 . [1]
f

71
2PBBBCPM7AS

14. At what position of the image in a simple microscope the magnifying power is (i) maximum
(ii) minimum?
Solution
(i) When the image is formed at the least distance of distinct vision. [1]
(ii) When the image is formed at infinity. [1]
15. Draw a neat labelled diagram of Cassegrain reflecting telescope.
Solution
Refer Module 7A: Diagram [1]
Labelling [1]
16. The intensities of two interfering beams in a Young’s double slit experiment are in the ratio 1 : 9.
Compare the intensities of the bright and dark fringes.
Solution
I1 1
=
I2 9
A 1
∴ 1 =
A2 9
(∵ I ∝ A 2 ) [1]
2
Ibright (A + A 2 )2
= 1 =
( 9+ 1 ) =
16 4
= [1]
2
(A1 − A 2 ) 2
Idark
( 9− 1) 4 1

17. Two towers on the top of two hills are 40 km apart. The line joining them passes 50 m above a hill
halfway between the towers. What is the longest wavelength of radio waves which can be sent between
the towers without appreciable diffraction effects?
Solution
(If there were to be no diffraction by the hill for the radio beams, the radial spread of the beam over the
hill 20 km away must not exceed 50 m.)
i.e. ZF (Fresnel’s distance) = 20 km = 20 × 103 m [1]
a = 50 meter
a2
ZF =
λ
a 2 50 × 50
 λ= = 3
= 1250 × 10−4 m [1]
ZF 20 × 10
Thus, the longest wavelength of radio waves is 0.125 m.
18. Two polaroids with their pass-axis aligned at 60° are placed in the path of a beam of unpolarised light of
intensity I0. What is the intensity of emerging light?
Solution
Intensity of emergent light from the first Polaroid, I1 = I0/2. [1]
Light of intensity (I0/2) falls on the second Polaroid with its pass-axis at 60°.
I I 1 I
∴ Intensity of light transmitted is given by I2 = I1 cos2 θ = 0 cos 2 60° = 0   = 0 [1]
2 2 4 8

72
2PBBBCPM7AS

PART - C
III. Answer any FIVE of the following questions [5 × 3 = 15]
19. Show that the focal length of a concave mirror is half of its radius of curvature.
Solution
Refer Module 7A
Diagram [1]
Derivation [2]
20. An object is placed at 5 cm in front of a concave mirror of radius of curvature 15 cm, find the position of
the image.
Solution
R 15
U = − 5 cm, f = = − cm, v = ? [1]
2 2
1 1 1
+ = [1]
v u f
1 1 2 1 2 1 −2 + 3 1
+ =− =− + = =+
v (−5) 15 v 15 5 15 15
v = 15 cm (virtual, erect image) [1]
21. Derive an expression for effective focal length of combination of two thin lenses kept in contact,
co-axially.
Solution
Refer Module 7A
Diagram + introduction [1]
Derivation: feq [1]
22. Draw a labelled diagram of Cassegrain Telescope. Write the expression for magnifying power.
Solution
Refer Module 7A
labelled Diagram [2]
Chapter 9.5
23. Discuss refraction of a plane wave at a plane surface and prove the laws of reflection using Huygen’s
principle.
Solution
Refer Module 7A
Diagram + introduction [1]
Derivation [1]
24. Obtain the expression for intensity of a fringe in a double slit interference pattern.
Solution
Refer Module 7A
Introduction + terms used [1]
 φ
Expression for intensity = 4I 0 cos2   [1]
2
25. What is resolving power of a microscope? Write the expression for limit of resolution and resolving
power of a microscope. Describe any one method to increase the resolving power of a microscope.
Solution
Refer Module 7A [1 + 1 + 1]

73
2PBBBCPM7AS

26. Define polarising angle. State and prove Brewster’s law.


Solution
Refer Module 7A
Definition [1]
Derivation n = tan θp [2]

PART - D
IV. Answer any TWO of the following questions [2 × 5 = 10]
27. Obtain the relation between refractive index of the material of a prism when the prism is placed in the
minimum deviation position.
Solution
Refer Module 7A
Diagram + Introduction [1]
D = i1 +i2 − A [1]
r1 + r2 = A [1]
Graph [1]
Final expression [1]
28. Derive the formula for refraction at a spherical surface when the surface is concave towards a denser
medium and a point object is in the rarer medium, forming a real image.
Solution
Refer Module 7A
Diagram + introduction [1]
Expression for i and r [2]
Final expression [2]
29. Obtain the expression for magnification due to a compound microscope in normal adjustment.
Solution
Refer Module 7A
Diagram + introduction [2]
Expression for m [3]

V. Answer any TWO of the following questions [2 × 5 = 10]


30. Obtain the expression for fringewidth in Young’s double slit experiment.
Solution
Refer Module 7A
Diagram + introduction [1]
Path difference [2]
Expression [2]
31. Draw a schematic diagram of the arrangement for diffraction at a single slit. Explain the formation of the
diffraction pattern.
Solution
Refer Module 7A
Diagram + introduction [1]
Central bright [l]
Minima condition [2]
Secondary Maxima condition [1]

74
2PBBBCPM7AS

32. Describe the construction and working of an astronomical telescope.


Solution
Refer Module 7A
Obtain the lens maker’s formula
Diagram (two) [2]
Derivation [3]

VI. Answer any THREE of the following questions [3 × 5 = 15]


33. A ray of light is incident on a glass slab at an angle of 45°. If refractive index of glass is 1.6, calculate
the angle of deviation at the first surface and critical angle.
Solution
i = 45°, n = 1.6
sin i
n= [1]
sin r
sin i sin 45° 0.707
sin r = = = = 0.4418
n 1.6 1.6
r = sin −1 (0.4418) = 26° 13′ [1]
Deviation, d = i − r = 45° − 26° 13′ =18° 47′ [1]
1
Critical angle: n = [1]
sin C
1 1
sin C = = = 0.625 ; C = sin −1 (0.625) = 38° 41′ [1]
n 1.6
Result: Deviation = 18° 47′; critical angle = 38° 41′
34. An equilateral glass prism made of a material of refractive index (3/2) is immersed in water of refractive
index (4/3). Calculate the angle of incidence of a monochromatic ray such that it is equal to angle of
emergence.
Solution
3 4
n g = , n w = , A = 60°, D = ?, i (for minimum deviation) = ?
2 3
3
n g  2  9
w ng = = = [1]
nw  4  8
 
3
When the angle of incidence is equal to the angle of emergence, the prism is in the minimum deviation
 A+D 
sin  
position, w n g =  2  [1]
A
sin  
2
 60° + D 
sin  
9
=  2  = 2sin  60° + D   sin  60°+ D  = 9 = 0.5625 [1]
   
8  60°   2   2  16
sin  
 2 
 60° + D  −1
  = sin (0.5625) = 34°13′
 2 
60° + D = 68°26′  D = 68°26′ − 60° = 8°26′ [1]

75
2PBBBCPM7AS

 A+D 
∴ i=  = 34°13′ [1]
 2 
Result: (i) Angle of minimum deviation = 8°26′ (ii) Angle of incidence = 34°13′
35. The apparent thickness of a plano-convex lens is measured with: (a) the plane face upper most and
(b) the convex face upper most. The values of the apparent thickness in the two cases are 0.02 m and
(1/45) m respectively. If the maximum thickness of the lens is 0.03 m, calculate its focal length.
Solution
(a) Refraction at the plane surface:
real thickness 0.03
RI, n = = = 1.5 [1]
apparent thickness 0.02 P
(b) Refraction at the convex surface:
1
u = −0.03 m, v = − m (virtual image), no = 1.5, ni = 1
45
n n n ~ n0
For refraction at the spherical surface of the lens, i − 0 = i [1]
v u R
1 1.5 1 − 1.5
Substituting the known values, we get − =
 1  −0.03 R
 − 
 45 
−0.5 −0.5
−45 + 50 =  5= R = – 0.1 m [1]
R R
For the plano-convex lens, R1 = ∞, R2 = R = −0.1 m
1  1 1 
Lens maker’s formula is = (n − 1)  − 
f  R1 R 2 
1 1 1  1 R1 R2
= (1.5 − 1)  −  = 5  f = = 0.2 m [2]
f  ∞ (−0.1)  5
Result: Focal length of the lens = 0.2 m.
36. An angular magnification (magnifying power) of 30 X is desired using an objective of focal length
1.25 cm and an eye-piece of focal length 5 cm. How will you set up the compound microscope?
Solution
Focal length of objective, fo = 1.25 cm
Focal length of eye-piece, fe = 5 cm
Least distance of distinct vision, D = 25 cm
Magnifying power = Angular magnification (m) = 30
D 25
The magnification produced by eye-piece m e = 1 + = 1 + =6 [1]
fe 5
The magnification produced by a microscope, m = mo × me, where, mo is the magnification produced by
objective lens. 30 = mo × 6 ∴ mo = 5

vo
Magnification due to the objective lens, m o =
uo
vo
−5 = (–ve sign shows that the image due to the objective lens is inverted w.r.t the objects)
uo
vo = − 5uo [1]

76
2PBBBCPM7AS

1 1 1
Using Lens formula for objective lens, = −
fo vo u o
1 1 1 6
= − =−
1.25 −5u o u o 5u o
6
From Eq. (1) we get u o = − × 1.25 = −1.5 cm [1]
5
v o = − 5uo = − 5 (−1.5) = 7.5 cm
Thus, the object should be placed at a distance of 1.5 cm from the objective lens to get the desired
1 1 1
magnification. By lens formula for eye-piece, = −
f e ve u e
1 1 1 1 1 6
= − =− − =− (∵ v e = −D = −25 cm)
u e ve f e 25 5 25
ue = − 4.17 cm [1]
The separation between objective and eye-piece
| v o | + | ue | = 4.17 + 7.5 = 11.67 cm [1]
Result: The object should be placed 1.5 cm from the objective to obtain the desired magnification.
The length of the microscope should be 11.67 cm.
37. In Young’s double-slit experiment using monochromatic light of wavelength λ, the intensity of light at
point on the screen where path difference is λ is K units. What is the intensity of light at a point where
path difference is λ/3?
Solution
Wavelength of light = λ. Let I1= I2 = I
2π 2π
When the path difference is λ, the phase difference φ = x λ = 2π [1]
λ λ
Resultant intensity, IR = I1 + I2 + 2 I1I 2 cos φ [1]
IR = I + I + 2 (I)(I) cos 2π = 2I + 2I = 4I = K (Given) …(1)
2 π λ 2π
When the path difference is λ/3, Phase difference = × = [1]
λ 3 3
In this condition, resultant intensity
2π  1  2π 1
I′R = I + I + 2 I2 cos = 2I + 2I  −  ∵ cos 3 = − 2  [1]
3  2  
K
I′R = I = [From Eq.(1)] [1]
4
λ K
Result: The intensity of light at a point where the path difference is .
3 4

***

77

You might also like